You are on page 1of 80

.

IASBABA’S PRELIMS TEST SERIS (AIPTS) 2024 – CURRENT AFFAIRS )(JULY 2023)

Q.1) Consider the following statements c) Both 1 and 2


about the PM Programme for Restoration, d) Neither 1 nor 2
Awareness, Nourishment, and
Amelioration of Mother Earth (PM-
PRANAM) Scheme: Q.3) Consider the following statements:

1. Its objective is to encourage the Statement-I :


balanced use of fertilizers Vande Bharat 2.0 trains contribute
in conjunction with biofertilizers towards achieving sustainable
and organic fertilizers. development goal 11.
2. It will be financed by the savings of Statement-II:
existing fertilizer subsidies under Vande Bharat 2.0 trains have a photo-
schemes run by the Department of catalytic ultraviolet air purification system
Fertilizers, Ministry of Chemicals & installed in the Roof-Mounted Package
Fertilizers. Unit (RMPU) for air purification.
3. The reduction in urea consumption
by a state will be compared to Which one of the following is correct in
its average consumption of urea respect of the above statements?
over the previous five years.
a) Both Statement-I and Statement-II
Choose the correct code: are correct and Statement-II is the
correct explanation for Statement-
a) 1 and 2 I
b) 2 and 3 b) Both Statement-I and Statement-II
c) 1 and 3 are correct and Statement-II is not
d) 1, 2 and 3 the correct explanation' for
Statement-I
c) Statement I is correct but
Q.2) Consider the following statements Statement II is incorrect
about the National Sickle Cell Anaemia d) Statement I is incorrect Statement
Elimination Mission: II is correct

1. It aims to eliminate sickle cell


disease as a public health problem Q.4) Consider the following statements:
in India by the year 2035.
2. It is implemented in all the states Statement-I :
and union territories across the Sulfur is a plant macronutrient that is
country. suitable for fertilizer coating.
Statement-II:
Choose the correct code: It is because of its ability to melt at a high
temperature.
a) 1 only
b) 2 only
.
IASBABA’S PRELIMS TEST SERIS (AIPTS) 2024 – CURRENT AFFAIRS )(JULY 2023)

Which one of the following is correct in Q.6) Consider the following statements:
respect of the above statements?
Statement-I :
a) Both Statement-I and Statement-II The “Parrot Lady of Khajuraho”
are correct and Statement-II is the sculpture was returned to India from
correct explanation for Statement- Britain.
I Statement-II:
b) Both Statement-I and Statement-II It was returned to India according to the
are correct and Statement-II is not UNESCO Convention of 1970.
the correct explanation' for
Statement-I Which one of the following is correct in
c) Statement I is correct but respect of the above statements?
Statement II is incorrect
a) Both Statement-I and Statement-II
d) Statement I is incorrect Statement
are correct and Statement-II is the
II is correct
correct explanation for Statement-
I
b) Both Statement-I and Statement-II
Q.5) Consider the following statements:
are correct and Statement-II is not
1. The Sanskrit language isincluded in the correct explanationfor
the eighth schedule of the Indian Statement-I
constitution. c) Statement I is correct but
2. Viswa Samskrita Dinam is an Statement II is incorrect
annual event celebrated d) Statement I is incorrect Statement
on 22nd August. II is correct
3. Sudharma is the only Sanskrit
newspaper in the world.
Q.7) Consider the following statements:
How many of the above statements are
correct? 1. Mata niPachedi is a handmade
textileof Gujarat.
a) Only one 2. Kanal Brass Set isa traditional
b) Only two musical instrument of Madhya
c) All three Pradesh.
d) None 3. Pithora is a ritualistic tribal folk art
of Karnataka.

How many of the above statements are


correct?

a) Only one
b) Only two
c) All three
.
IASBABA’S PRELIMS TEST SERIS (AIPTS) 2024 – CURRENT AFFAIRS )(JULY 2023)

d) None a) Only one


b) Only two
Q.8) Sozni is a popular needlepoint c) All three
embroidery technique of? d) None

a) Himachal Pradesh
b) Jammu and Kashmir Q.11) Consider the following statements
c) Uttarakhand regarding Siachen Glacier:
d) a), b), and c)
1. It is under the administration of
Q.9) Consider the following statements Pakistan, post Operation
regarding the recent report by Meghdoot.
the Association of Democratic Reforms 2. It is the world’s longest glacier in
(ADR) on donations made to the political non-polar areas.
parties: 3. It is the world's highest battlefield.
4. It is the place where the Nubra
1. The donations made to political River originates.
parties by the corporate sector are
higher than the electoral bonds. How many of the above statements are
2. Both national parties and regional correct?
parties experienced increased
donations from electoral bonds. a) Only one
b) Only two
Choose the correct code: c) Only three
d) All four
a) 1 only
b) 2 only
c) Both 1 and 2 Q.12) Consider the following statements:
d) Neither 1 nor 2
Statement-I :
Phosphine can be made only industrially
Q.10) Which of the following cases can be and not naturally.
taken up by the National Human Rights Statement-II:
Commission (NHRC): It can be manufactured industrially
from white phosphorus by hydrolysis with
1. Abuse of the legal system in the an alkali metal hydroxide or an aqueous
trafficking of girls acid.
2. Rape of a visually impaired girl
3. Death of a boy in the observation Which one of the following is correct in
home respect of the above statements?

Choose the correct code: a) Both Statement-I and Statement-II


are correct and Statement-II is the
.
IASBABA’S PRELIMS TEST SERIS (AIPTS) 2024 – CURRENT AFFAIRS )(JULY 2023)

correct explanation for Statement- 2. It does not apply to the Ram


I Janmabhoomi-Babri Masjid case,
b) Both Statement-I and Statement-II and any suit, appeal, or proceeding
are correct and Statement-II is not relating to it.
the correct explanation for 3. It prescribes a punishment of a
Statement-I maximum of three years
c) Statement I is correct but imprisonment along with a fine for
Statement II is incorrect contravening the provisions of the
d) Statement I is incorrect Statement Act.
II is correct
How many of the above statements are
correct?
Q.13) Consider the following pairs
a) Only one
Dam River b) Only two
c) All three
d) None
1. Rana Pratap Chambal River
Sagar Dam
2. Tajewala Yamuna River
Q.15) Consider the following statements
Barrage Dam
about Hampi:
3. Rajghat Dam Ken River
1. It is located in the Tungabhadra
4. Gangau Dam Betwa River basin in Bellary District, Central
Karnataka.
2. It was the capital city during
Choose the correct code: Sangama, Tuluva, Cholas, and
Aravidu.
a) One pair
3. It is a UNESCO World Heritage
b) Two pairs
Site and is called the “World’s
c) Three pairs
Largest Open-air Museum”.
d) Four pairs
How many of the above statements are
correct?
Q.14) Consider the following statements
regarding the provisions of the Places of a) Only one
Worship Act of 1991: b) Only two
c) All three
1. It declares that the religious d) None
character of a place of worship
shall continue to be the same as it
existed on August 15, 1947.
.
IASBABA’S PRELIMS TEST SERIS (AIPTS) 2024 – CURRENT AFFAIRS )(JULY 2023)

Q.16) Consider the following statements Q.18) Consider the following statements
about SAGAR SAMPARK: about Perseverance Rover:

1. It is an indigenous Differential 1. Its primary objective is to look for


Global Navigation Satellite System signs of ancient microbial life on
(DGNSS) launched by the Indian Venus.
Space Research Organisation. 2. It will manufacture molecular
2. It provides safe navigation, oxygen on Mars using MOXIE.
reducing the risk of collisions, 3. It has the first-ever helicopter to
groundings, and accidents at sea. fly on Mars called Ingenuity.

Choose the correct code: Choose the correct code:

a) 1 only a) 1 only
b) 2 only b) 2 and 3
c) Both 1 and 2 c) 1 and 3
d) Neither 1 nor 2 d) 1, 2 and 3

Q.17) It is nestled in the Aravali hills in Q.19)The GrossNon-Performing Assets


Rajasthan with mountains, grasslands, dry (GNPA) of the country’s scheduled
deciduous forests, and cliffs. It was commercial banks as per recently released
declared a wildlife sanctuary in 1955 and a Financial Stability Report of RBI in March
tiger reserve in 1978. It has a blend 2023?
of tropical dry deciduous and tropical
thorn forests. Kankarwadi Fort is located a) 8.7%
in the center of the reserve. It houses b) 3.9%
the Lord Hanuman temple and Neelkanth c) 4.3%
temple. d) 3.4%
The above paragraph refers to which of
the following reserve?
Q.20) Consider the following statements
a) Ranthambore Tiger Reserve about Section 144 of the Code of Criminal
b) Mukundara Tiger Reserve Procedure (CrPC) of 1973:
c) Ramgarh Tiger Reserve
d) Sariska Tiger Reserve 1. It places restrictions on handling or
transporting any kind of weapon in
the given jurisdiction.
2. It empowers a district
magistrate specially empowered
by the state government to issue
orders.
.
IASBABA’S PRELIMS TEST SERIS (AIPTS) 2024 – CURRENT AFFAIRS )(JULY 2023)

3. The order under this section can 3. He opposed the British in response
remain in force for a period of to the Madras Forest Act of 1882
more than twelve months. in the backdrop of the Swadeshi
movement.
Choose the correct code:
How many of the above statements are
a) 1 and 2 correct?
b) 2 and 3
c) 1 and 3 a) Only one
d) 1, 2 and 3 b) Only two
c) All three
d) None
Q.21) Consider the following statements
about the National Maritime Heritage
Complex: Q.23) Consider the following statements
about India Mobile Congress (IMC):
1. It is being constructed by the
Ministry of Ports, Shipping, and 1. The theme of the India Mobile
Waterways. Congress-2023 is “Global Digital
2. It is being constructed in the Indus Innovation”.
Valley civilization region of 2. It is jointly organized by
Rangpur. the Department of
3. It will house Asia’s biggest Telecommunications (DoT) and the
underwater marine museum. Cellular Operators Association of
India (COAI).
How many of the above statements are
correct? Choose the correct code:

a) Only one a) 1 only


b) Only two b) 2 only
c) All three c) Both 1 and 2
d) None d) Neither 1 nor 2

Q.22) Consider the following statements Q.24) Consider the following statements
about Alluri Sitarama Raju: about National Bank for Agriculture and
Rural Development (NABARD):
1. He is considered as Manyam
Veerudu for his bravery and 1. It is a statutory body formed in
sacrifice. 1982 under the Reserve Bank of
2. He led a guerrilla campaign in India of 1934 Act.
the Eastern Ghats region of Andhra 2. It deals with the operations of
Pradesh. giving credit for agriculture and
.
.
.
.
IASBABA’S PRELIMS TEST SERIS (AIPTS) 2024 – CURRENT AFFAIRS )(JULY 2023)

b) 2 and 3 Q.39) Consider the following statements


c) 1 and 3 about LCA Tejas:
d) 1, 2 and 3
1. It is a multipurpose supersonic
aircraft.
Q.37) Consider the following statements 2. It is manufactured by Hindustan
about Indian Grey Hornbill: Aeronautics Limited (HAL).

1. They are arboreal and found only Choose the correct code:
in the foothills of the Southern
a) 1 only
Himalayas.
b) 2 only
2. They are listed as endangered on
c) Both 1 and 2
the IUCN List.
d) Neither 1 nor 2
3. They are the smallest in the
Hornbill family.

How many of the above statements are Q.40) Dinutuximab (Qarziba) is used to
correct? treat:

a) Only one a) Leprosy


b) Only two b) Dengue
c) All three c) Chikungunya
d) None d) Cancer

Q.38) Consider the following statements Q.41) Ubinas Volcano is located in:
about SEBI’s Complaints Redress System
a) Chicago
(SCORES) platform:
b) Peru
1. It is launched by the Reserve Bank c) Chile
of India. d) Sudan
2. It does not deal with complaints
against companies.
3. It deals with complaints under the Q.42) Which of the following are included
Companies Act, 2013. underGramodyog Vikas Yojana (GVY)?

How many of the above statements are 1. Wellness & Cosmetics Industry
correct? (WCI)
2. Handmade Paper, Leather, and
a) Only one Plastic Industry (HPLPI)
b) Only two 3. Agro-based and Food Processing
c) All three Industry (ABFPI)
d) None 4. Mineral-based Industry (MBI)
.
IASBABA’S PRELIMS TEST SERIS (AIPTS) 2024 – CURRENT AFFAIRS )(JULY 2023)

Choose the correct code: Q.45) Consider the following statements


about Similipal National Park:
a) Only one
b) Only two 1. It is a national park and a tiger
c) Only three reserve located in Assam.
d) All four 2. It is surrounded by the twin peaks
of Khairiburu and Meghashini.
3. Its vegetation is a mix of deciduous
Q.43) The World Investment Report is and semi-evergreen forests.
published by the? 4. It houses tribes like Kolha,
Santhala, Bhumija, Bhatudi, and
a) World Economic Forum Gondas.
b) World Trade Organisation
c) United Nations Conference on How many of the above statements are
Trade and Development (UNCTAD) correct?
d) None
a) Only one
b) Only two
Q.44) Consider the following statements c) Only three
with reference to the World Investment d) All four
Report 2023:
Q.46) Consider the following statements
1. India and ASEAN registered a about James Webb Space Telescope:
decrease in foreign direct
investment (FDI). 1. It is an infrared observatory
2. China is the second-largest FDI orbiting the Earth.
host country in the world. 2. It is launched by the Indian Space
3. FDI registered a decline in Gulf Research Organisation (ISRO).
countries and the least developed 3. It aims to study every phase in the
countries. history of our universe.

How many of the above statements are How many of the above statements are
correct? correct?

a) Only one a) Only one


b) Only two b) Only two
c) All three c) All three
d) None d) None
.
IASBABA’S PRELIMS TEST SERIS (AIPTS) 2024 – CURRENT AFFAIRS )(JULY 2023)

Q.47) Consider the following statements d) Neither 1 nor 2


about Special Window for Funding Stalled
Affordable and Middle-Income Housing
Projects (SWAMIH) Fund: Q.49) Consider the following statements
about Gucchi:
1. It is a social impactfund specifically
formed for completing stressed 1. It is a species of fungus.
and stalled residential projects. 2. It is found in temperate forests.
2. To be eligible for funding, projects 3. It is rich in antioxidant and
must be registered under the Real antimicrobial properties.
Estate (Regulation and
Development) Act (RERA) 2016. Choose the correct code:
3. It is sponsored by the Ministry of
a) 1 and 2
Housing and Urban Affairs,
b) 62 and 3
Government of India.
c) 1 and 3
How many of the above statements are d) 1, 2 and 3
correct?

a) Only one Q.50) Consider the following statements


b) Only two about Wildlife Crime Control Bureau
c) All three (WCCB):
d) None
1. It is a statutory body established
under the Environment Protection
Q.48) Consider the following statements Act of 1986.
about the One Health Priority Research 2. Its objective is to combat
Agenda on Antimicrobial Resistance: organized wildlife crime in the
country.
1. It is launched by the World Health
Organization (WHO) and the World Choose the correct code:
Organisation for Animal Health
a) 1 only
(WOAH).
b) 2 only
2. It is based on pillars such as
c) Both 1 and 2
integrated surveillance,
d) Neither 1 nor 2
interventions, behavioural insights,
and change.
Q.51) Consider the following statements:
Choose the correct code:
1. Global Gibbon Network is
a) 1 only established by the United Nations
b) 2 only Environment Programme.
c) Both 1 and 2
.
IASBABA’S PRELIMS TEST SERIS (AIPTS) 2024 – CURRENT AFFAIRS )(JULY 2023)

2. Hoolock Gibbon is the only ape 2. It will be implemented by a special


found in India and is the state purpose vehicle (SPV) completely
animal of Sikkim. owned by the centre through the
3. Both Western Hoolock Gibbon and Ministry of Textiles.
Eastern Hoolock Gibbon are
protected under Schedule I of the Choose the correct codes:
Wildlife Protection Act of 1972.
a) 1 only
How many of the above statements are b) 2 only
correct? c) Both 1 and 2
d) Neither 1 nor 2
a) Only one
b) Only two Q.55) Consider the following statements:
c) All three
d) None Statement I:
Urbanization and concrete structures in
Q.52) Purana cheeveda commonly found cities cause heat waves.
in several parts of South India is a/an? Statement II:
Because concrete and asphalt in cities
a) Frog absorb and retain heat leading to
b) Fish increased temperatures contributing to
c) Snail heat waves.
d) Insect
Which one of the following is correct in
Q.53) The strategic roadmap “Horizon respect of the above statements?
2045” is between which of the following
a) Both Statement-I and Statement-II
countries?
are correct and Statement-II is the
a) United States and Germany correct explanation for Statement-
b) Australia and Japan I
c) India and France b) Both Statement-I and Statement-II
d) China and Sri Lanka are correct and Statement-II is not
the correct explanation' for
Q.54) Consider the following statements Statement-I
about PM Mega Integrated Textile c) Statement I is correct but
Regions and Apparel (PM MITRA) Parks: Statement II is incorrect
d) Statement I is incorrect Statement
1. It will help in creating world-class II is correct
industrial infrastructure by
encouraging investment,
innovation, and job creation in the
textile sector.
.
IASBABA’S PRELIMS TEST SERIS (AIPTS) 2024 – CURRENT AFFAIRS )(JULY 2023)

Q.56) Consider the following statements a) Only one


about the Comprehensive and Progressive b) Only two
Agreement for Trans-Pacific Partnership c) All three
(CPTPP): d) None

1. It is a free trade agreement that Q.58) Consider the following statements


succeeded the Trans-Pacific regarding Pre-arrest Bail:
Partnership (TPP).
2. All members of the Asia-Pacific 1. It is granted under section 438 of
Economic Cooperation (APEC) are the Code of Criminal Procedure of
members of the CPTPP. 1973.
3. The United Kingdom is the first 2. It is issued only by the Supreme
European country and the first Court and High Court.
new member of the bloc since its 3. The person can seek pre-arrest bail
inception. only for non-bailable offences.

How many of the above statements are How many of the above statements are
correct? correct?

a) Only one a) Only one


b) Only two b) Only two
c) All three c) All three
d) None d) None

Q.57) Consider the following statements Q.59) Consider the following statements:
regarding the National Multidimensional
Poverty Index: A Progress Review 2023 by Statement I:
NITI Aayog: If two countries have a Local Currency
Settlement System (LCSS), trade between
1. Bihar topped the list with the them won’t be affected due to sanctions
highest percentage of people who and global political issues.
are living in multidimensional Statement II:
poverty. LCSS allows trading entities from two
2. Uttar Pradesh registered the countries to make payments in their
largest decline in number of poor national currencies.
people escaping multidimensional
poverty. Which one of the following is correct in
3. The rural areas witnessed the respect of the above statements?
fastest decline in poverty than
a) Both Statement-I and Statement-II
urban areas.
are correct and Statement-II is the
How many of the above statements are
correct explanation for Statement-
correct?
I
.
IASBABA’S PRELIMS TEST SERIS (AIPTS) 2024 – CURRENT AFFAIRS )(JULY 2023)

b) Both Statement-I and Statement-II How many of the above statements are
are correct and Statement-II is not correct?
the correct explanation' for
Statement-I a) Only one
c) Statement I is correct but b) Only two
Statement II is incorrect c) All three
d) Statement I is incorrect Statement d) None
II is correct
Q.62) Which of the following disease is
Q.60) Consider the following statements treated using the drug, ‘Leqembi’?
about the Kuki tribe:
a) Dementias
1. They are present in all Northeast b) Parkinson’s disease
Indian states except Arunachal c) Epilepsy
Pradesh. d) Alzheimer’s disease
2. Lal is known as their village chief
and has their customary laws. Q.63) Consider the following statements
3. They resisted British hegemony about Electronic Bank Realization
before India’s independence. Certificate (e-BRC):

How many of the above statements are 1. It is proof of export issued by


correct? banks.
2. It is essential to avail export
a) Only one incentives from the government.
b) Only two
c) All three Choose the correct codes:
d) None
a) 1 only
b) 2 only
Q.61) Consider the following statements
c) Both 1 and 2
about Black Sea Grain Initiative:
d) Neither 1 nor 2
1. It was set up to resume vital food
and fertilizer exports from Q.64) Consider the following statements
Ukraine to the rest of the world. about the Enforcement Directorate (ED):
2. It was brokered between Russia
1. It is a multi-disciplinary
and Ukraine by the Food and
organization that works under the
Agriculture Organization and India.
Department of Revenue, Ministry
3. It is implemented by the Joint
of Finance.
Coordination Centre which is
2. It enforces the provisions of the
hosted in Istanbul.
Foreign Exchange Management
Act (FEMA) of 1999 and the
.
IASBABA’S PRELIMS TEST SERIS (AIPTS) 2024 – CURRENT AFFAIRS )(JULY 2023)

Prevention of Money Laundering Land Records Modernization


Act of (PMLA) 2002. Programme (DILRMP).
3. Its officers are recruited from 2. DILRMP is a central sector
various investigation agencies like scheme being implemented by the
Indian Revenue Services (IRS), Department of Land
Indian Police Services (IPS), and Resources under the Ministry of
Income Tax Officers. Home Affairs.

How many of the above statements are Choose the correct codes:
correct?
a) 1 only
a) Only one b) 2 only
b) Only two c) Both 1 and 2
c) All three d) Neither 1 nor 2
d) None
Q.67) Consider the following statements
Q.65) Consider the following statements about the Bay of Bengal Initiative for
about Steel Slag Road Technology: Multi-Sectoral Technical and Economic
Cooperation (BIMSTEC):
1. It uses the waste produced during
steel production to build robust 1. It is a regional organization that
and durable roads. was established by the Bangkok
2. It increases the reliance on natural Declaration.
resources and is unsustainable in 2. All its member countries are from
the long term. Asia and Africa.
3. It plays a very important role in 3. Its secretariat is in Dhaka,
fulfilling the mission of waste to Bangladesh.
wealth in India.
How many of the above statements are
How many of the above statements are correct?
correct?
a) Only one
a) Only one b) Only two
b) Only two c) All three
c) All three d) None
d) None
Q.68) Consider the following statements
Q.66) Consider the following statements: about Khashaba Dadasaheb Jadhav:
1. Bhoomi Samman Awards aims to 1. He is an Indian wrestler who is
recognise outstanding nicknamed ‘Pocket Dynamo’.
performance in the
implementation of the Digital India
.
IASBABA’S PRELIMS TEST SERIS (AIPTS) 2024 – CURRENT AFFAIRS )(JULY 2023)

2. He was the first athlete from tigers. It has culturally significant


independent India to win an monuments like Bhimlat.
individual medal in the Olympics.
3. He provided shelter to The above paragraph describes which of
revolutionaries during the Non- the following tiger reserve?
Cooperation Movement.
a) Ranthambore Tiger Reserve
How many of the above statements are b) Sariska Tiger Reserve
correct? c) Mukundra Tiger Reserve
d) Ramgarh Vishdhari Tiger Reserve
a) Only one
b) Only two Q.71) Consider the following pairs
c) All three Intercontinental Ballistic Missile (ICBM)
d) None Country

Q.69) Consider the following statements 1. Dongfeng-41 - Russia


about Gingee Venkataramana Temple: 2. RS-24 Yars - China
3. TridentII - United States
1. It was built in the 16th century by 4. Hwasong-18 - North Korea
Muthyala Nayaka in Tail Nadu.
2. Its entrance gateway depicts How many given pairs are correctly
Ramayana scenes, Vishnu's matched?
incarnations, and the Samudra
a) One pair
Manthan.
b) Two pairs
3. Its premises have a tank called
c) Three pairs
Chettikulam which is fed by
d) Four pairs
perennial springs and do not dry
even in summer.
Q.72) Consider the following statements
How many of the above statements are about the Commission on Genetic
correct? Resources for Food and Agriculture
(CGRFA):
a) Only one
b) Only two 1. It is the only permanent
c) All three intergovernmental body focused
d) None on conserving all types of
biodiversity for food and
Q.70) It is the fourth tiger reserve in agriculture.
Rajasthan, India. It comprises hilly dry 2. Its membership is open to all
deciduous forests consisting of Dhok, members of the World Health
Khair and Salar on Vindhyan formations. It Organization (WHO).
plays a critical role in the movement of
.
IASBABA’S PRELIMS TEST SERIS (AIPTS) 2024 – CURRENT AFFAIRS )(JULY 2023)

3. It meets regularly to address Q.75) Consider the following statements


policies on genetic resources for about the Gomti River:
food and agriculture.
1. It is a tributary of the Brahmaputra
How many of the above statements are River.
correct? 2. It originates near Mainkot,
from Gomat Taala Lake.
a) Only one 3. Lucknow, the capital of Uttar
b) Only two Pradesh is located on its banks.
c) All three 4. It meets the Ganges River at Kaithi,
d) None Ghazipur.

Q.73) Which of the following fish is known How many of the above statements are
as ‘Mosquito Fish’ as it combats diseases correct?
like malaria and dengue?
a) Only one
a) Blue-finned Mahseer b) Only two
b) Garra Laishrami c) Only three
c) Hilsa Fish d) All four
d) Gambusia Affinis
Q.26) Consider the following statements:
Q.74) Which of the following are the
advantages of External Commercial Statement I:
Borrowing (ECB)? Advance Authorisation Scheme helps in
making India’s products competitive in the
1. It provides an opportunity to global market.
borrow large volumes of funds. Statement II:
2. It reduces the company’s rank Advance Authorization Scheme exempts
given by the rating agency. the basic customs duty, education cess,
3. It does not dilute the stake in the social welfare cess, anti-dumping duty,
company. countervailing duty, and safeguard duties.

How many of the above statements are Which one of the following is correct in
correct? respect of the above statements?

a) Only one a) Both Statement-I and Statement-II


b) Only two are correct and Statement-II is the
c) All three correct explanation for Statement-
d) None I
b) Both Statement-I and Statement-II
are correct and Statement-II is not
the correct explanation' for
Statement-I
.
IASBABA’S PRELIMS TEST SERIS (AIPTS) 2024 – CURRENT AFFAIRS )(JULY 2023)

c) Statement I is correct but c) All three


Statement II is incorrect d) None
d) Statement I is incorrect Statement
II is correct Q.79) Consider the following statements
about Gharials:
Q.77) Consider the following statements:
1. They are an indicator species
1. Bosporus Strait connects the Black found in the tributaries of River
Sea to the Sea of Marmara. Ganga.
2. The Strait of the 2. They are listed as critically
Dardanelles connects the Sea of endangered on the IUCN Red List.
Marmara to the Aegean Sea. 3. They are protected under Schedule
3. Kerch Strait connects the Black Sea I of the Wildlife Protection Act of
to the Sea of Azov. 1972.

How many of the above statements are How many of the above statements are
correct? correct?

a) Only one a) Only one


b) Only two b) Only two
c) All three c) All three
d) None d) None

Q.78) Consider the following statements Q.80) Consider the following statements
about the Index of Industrial Production regarding Rudragiri Hillock:
(IIP):
1. It has prehistoric rock paintings
1. It is published annually by the from the Mesolithic period located
National Statistical Office (NSO) in Karnataka.
under the Ministry of Statistics and 2. It is the first cave representing
Programme Implementation. murals of battle between the
2. It indicates the relative change Vanara brothers, Vali, and Sugriva.
over time in the volume of 3. It has a sketch of Hanuman,
production in the industrial sector. accompanied by sacred symbols of
3. Refinery products constitute the the conch and the fire altar.
highest percentage among the
eight core industries. Choose the correct code:

How many of the above statements are a) 1 and 2


correct? b) 3 only
c) 2 and 3
a) Only one d) 1 only
b) Only two
.
IASBABA’S PRELIMS TEST SERIS (AIPTS) 2024 – CURRENT AFFAIRS )(JULY 2023)

Q.81) Consider the following statements 1. Dairy Processing


about PM SVANidhi Scheme: 2. Meat Processing
3. Animal Feed Manufacturing
1. It provides collateral-free loans up 4. Breed Improvement Technology
to INR 50,000/- to street vendors 5. Veterinary Vaccine and
for one year tenure. Drugs Production
2. It is implemented by the Small 6. Animal Waste to Wealth
Industries Development Bank of Management
India (SIDBI).
3. An interest subsidy of 7% per Choose the correct code:
annum is provided on timely/ early
repayment of the loan. a) 1, 2, 4 and 5 only
b) 3, 4, 5 and 6 only
How many of the above statements are c) 1, 2, 3 and 5 only
correct? d) All of the above

a) Only one Q.84) Consider the following statements


b) Only two about Bacteriophages:
c) All three
d) None 1. They are viruses that infect and
replicate only in bacterial cells.
Q.82) Consider the following statements 2. They are vectors for horizontal
about picolinic acid: gene transfer.
3. They are found only in soil.
1. It is a pyridinemonocarboxylic acid.
2. It helps in the absorption of zinc Choose the correct code:
from our gut.
3. It disrupts the entry of enveloped a) 1 and 2
viruses into the host’s cell and b) 2 and 3
prevents infection. c) 1 and 3
d) 1, 2 and 3
How many of the above statements are
correct? Q.85) Consider the following statements
about DPT3 Vaccine:
a) Only one
b) Only two 1. It provides protection against
c) All three three severe viral diseases -
d) None Diphtheria, Tetanus, and Pertussis.
2. All children should get five doses
Q.83) Which of the following activities are of the DTP vaccine at 6 weeks, 10
eligible for benefits under the Animal weeks, 14 weeks, 16-18 months,
Husbandry Infrastructure Development and 5 years.
Fund (AHIDF)?
.
IASBABA’S PRELIMS TEST SERIS (AIPTS) 2024 – CURRENT AFFAIRS )(JULY 2023)

3. A child who had a life-threatening 3. Phishing is an attack meant to shut


allergic reaction after a dose of down a machine or
DTP vaccination should not get network, making it inaccessible to
another dose. its intended users.
4. Denial of Service is attempting
How many of the above statements are to obtain sensitive information for
correct? fraudulent activities, by disguising
oneself as an authentic entity via
a) Only one
electronic communication.
b) Only two
c) All three How many of the above statements are
d) None correct?

Q.86) Which of the following are the a) Only one


benefits of hydrogen as a fuel? b) Only two
c) Only three
1. It is environmentally d) All four
friendly and non-toxic.
2. It occurs in deposits or reserves Q.88) The Chicago Convention of 1944 is
like fossil fuel. related to the:
3. It is produced using biological
reactions. a) World Health Organisation (WHO)
4. It is more efficient than diesel or b) Food and Agriculture Organisation
gas. (FAO)
c) International Monetary Fund (IMF)
Choose the correct code: d) International Civil Aviation
Organization (ICAO)
a) Only one
b) Only two
Q.89) Consider the following statements
c) Only three
about India Climate Energy Dashboard
d) All four
(ICED) 3.0:

Q.87) Consider the following statements: 1. It is India’s one-stop platform for


near real-time data on the energy
1. Malware is any kind of software
sector, climate, and related
that is designed to cause
economic datasets.
damage to a single computer,
2. It is released by the Ministry of
server, or computer network.
Environment, Forest, and Climate
2. Spoofing involves impersonating a
Change.
bank’s URL with a website that is
quite similar to the original one Choose the correct code:
and has similar functions as well.
a) 1 only
.
IASBABA’S PRELIMS TEST SERIS (AIPTS) 2024 – CURRENT AFFAIRS )(JULY 2023)

b) 2 only Q.92) Consider the following statements


c) Both 1 and 2 about Silver cockscomb:
d) Neither 1 nor 2
1. It is a weed that attracts insects,
Q.90) Consider the following statements caterpillars, worms, and moths
about National Dairy Development Board that can harm crops.
(NDDB): 2. It is used as a nutritious leafy
green vegetable by the Soliga
1. It is a statutory body initially tribe.
registered as a society under the 3. It belongs to the Amaranthaceae
Societies Act 1860. family, which includes
2. It works under the Ministry of economically important plants like
Agriculture and Farmers Welfare. spinach and beetroot.
3. It launched the world’s largest
dairy development program, Choose the correct code:
Operation Flood.
a) 1 and 2
How many of the above statements are b) 2 and 3
correct? c) 1 and 3
d) 1, 2 and 3
a) Only one
b) Only two Q.93) Recently in the news, Ludwigia
c) All three peruviana is a:
d) None
a) Medicinal plant
Q.91) Which of the following are b) Frog species
restrictions for moving an adjournment c) Invasive weed
motion? d) Fish species

1. It must not cover more than one Q.94) Consider the following statements
matter. about Malati Devi Choudhury:
2. It must be restricted to a specific
matter of recent occurrence. 1. She organised the ‘Krusaka
3. It must not raise a question of Andolana’ as part of the freedom
privilege. struggle against the zamindars and
moneylenders.
How many of the above statements are 2. She organised the Utkal Congress
correct? Samajvadi Karmi Sangh and the
Bajiraut Chhatravas.
a) Only one
b) Only two Choose the correct code:
c) All three
d) None a) 1 only
.
IASBABA’S PRELIMS TEST SERIS (AIPTS) 2024 – CURRENT AFFAIRS )(JULY 2023)

b) 2 only Q.98) Consider the following statements


c) Both 1 and 2 regarding the State of Climate in Asia
d) Neither 1 nor 2 2022 Report: released by the World
Meteorological Organisation (WMO).
Q.95) Consider the following statements
about African Union: 1. The number of disaster events in
Asia decreased when compared to
1. It is a successor to the previous year.
the Organization of African Unity. 2. Pakistan suffered the most severe
2. All its 55 member countries are economic losses that occurred due
only from the African continent. to floods in the Asia region.
3. It is headquartered in Cape Town,
South Africa. Choose the correct code:

How many of the above statements are a) 1 only


correct? b) 2 only
c) Both 1 and 2
a) Only one d) Neither 1 nor 2
b) Only two
c) All three Q.99) Parachik Glacier is located in?
d) None
a) Denmark
Q.96) Consider the following statements b) India
about Gulf Stream: c) Finland
d) Antarctica
1. It is a strong cold ocean current.
2. It carries water from the Gulf of Q.100) Consider the following statements
Mexico into the Atlantic Ocean. about Colombo Security Conclave:

Choose the correct code: 1. It is a trilateral airforce security


grouping among member
a) 1 only countries.
b) 2 only 2. Its members are India, Sri Lanka,
c) Both 1 and 2 Maldives, and Mauritius.
d) Neither 1 nor 2
Choose the correct code:
Q.97) Batagaika Crater, the world’s
largest permafrost crater is located in: a) 1 only
b) 2 only
a) Norway c) Both 1 and 2
b) Japan d) Neither 1 nor 2
c) Iceland
d) Russia
.
IASBABA’S PRELIMS TEST SERIES (AIPTS) 2024 – CURRENT AFFAIRS (JULY 2023) SOLUTIONS

Q.1) Solution (a)

Statement Analysis:

• The PM Programme for Restoration, Awareness, Nourishment, and Amelioration of


Mother Earth (PM-PRANAM) Scheme's objective is to encourage the balanced use of
fertilizers in conjunction with biofertilizers and organic fertilizers and to reduce
the subsidy burden on chemical fertilizers, which was around Rs 2.25 lakh crores in
2022-2023.Hence statement 1 is correct.
• It will be financed by the savings of existing fertilizer subsidies under schemes run by
the Department of Fertilizers, Ministry of Chemicals & Fertilizers.Hence statement 2
is correct.
• The Centre will provide 50% of the subsidy savings to the states as a grant. Out of
the grant, 70% can be used to create assets related to the technological adoption of
alternative fertilizers and production units at various levels. The remaining 30% can
be used to reward and encourage farmers, panchayats, and other stakeholders
involved in fertilizer reduction and awareness generation.
• The reduction in urea consumption by a state will be compared to its average
consumption of urea over the previous three years. This calculation will determine
the eligibility for subsidy savings and grants. Hence statement 3 is incorrect.

Note:
• Biofertilizer consists of a carrier medium rich in live microorganisms. When applied
to seed, soil, or living plants, it increases soil nutrients or makes them biologically
available.
• Biofertilizers contain different types of fungi, root bacteria, or other microorganisms.
They form a mutually beneficial or symbiotic relationship with host plants as they
grow in the soil.
Source: https://www.thestatesman.com/what-is/what-is-pm-pranam-yojana-and-what-are-
its-benefits-1503196059.html

Q.2) Solution (d)

Statement Analysis:

• Sickle cell disease (SCD) is a chronic single-gene disorder causing a debilitating


systemic syndrome characterized by chronic anemia, acute painful episodes, organ
infarction, and chronic organ damage and by a significant reduction in life
expectancy.
• The National Sickle Cell Anaemia Elimination Mission aims to improve the care of all
sickle cell disease patients for a better future and to lower the prevalence of the
disease through a multi-faced coordinated approach towards screening and
awareness strategies.
.
IASBABA’S PRELIMS TEST SERIES (AIPTS) 2024 – CURRENT AFFAIRS (JULY 2023) SOLUTIONS

• It aims to eliminate sickle cell disease as a public health problem in India by 2047.
Hence statement 1 is incorrect.
• It is implemented in 17 high-focus states across the country, this program aims to
improve the care and prospects of all sickle cell disease patients while reducing the
prevalence of the disease.
• The 17 states are- Gujarat, Maharashtra, Rajasthan, Madhya Pradesh, Jharkhand,
Chhattisgarh, West Bengal, Odisha, Tamil Nadu, Telangana, Andhra Pradesh,
Karnataka, Assam, Uttar Pradesh, Kerala, Bihar, and Uttarakhand. Hence statement
2 is incorrect.
Source:
https://pib.gov.in/PressReleasePage.aspx?PRID=1936735#:~:text=About%20National%20Sic
kle%20Cell%20Anaemia,tribal%20populations%20of%20the%20country

Q.3) Solution (a)

Statement Analysis:

• Sustainable Development Goal 11 emphasizes the need to make cities and human
settlements inclusive, safe, resilient, and sustainable. It addresses issues such as
urbanization, air pollution, waste management, and access to green spaces. Creating
sustainable cities and communities is crucial for environmental sustainability, as the
majority of the global population resides in urban areas. Hence statement 1 is
correct.
• The Vande Bharat Express 2.0 offers a myriad of superior and aircraft-like travelling
experiences.
• In the new design of Vande Bharat Express, a photo-catalytic ultraviolet air
purification system is installed in the Roof-Mounted Package Unit (RMPU) for air
purification. Hence statement 2 is correct.
• It is equipped with advanced state-of-the-art safety features including an
indigenously developed Train Collision Avoidance System - KAVACH.
Source: https://newsonair.com/2023/07/04/pm-modi-to-flag-off-25th-vande-bharat-
express-from-gorakhpur-to-lucknow-on-july-7/

Q.4) Solution (a)


Statement Analysis:

• Sulfur is a low-cost plant macronutrient and is suitable for fertilizer coating. Hence
statement 1 is correct.
• It is because of its ability to melt at a high temperature (at about 156°C),
degradability, and improved dry matter production. Hence statement 2 is correct.
.
IASBABA’S PRELIMS TEST SERIES (AIPTS) 2024 – CURRENT AFFAIRS (JULY 2023) SOLUTIONS

• Urea coated with sulfur helps in the slow release of nitrogen, therefore it reduces
nitrogen losses and increases nitrogen availability and uptake.
Source: https://www.thehindu.com/news/national/our-attempt-is-to-end-import-
dependency-on-urea-by-2025-and-replace-it-with-nano-urea-and-other-alternate-forms-of-
urea/article67064913.ece

Q.5) Solution (c)

Statement Analysis:

• Sanskrit is one of the oldest languages in the world. It belongs to the Indo-Aryan
language group and was used to write Vedas.
• The Eighth Schedule to the Constitution of India lists the official languages of the
Republic of India. Part XVII, Articles 343 to 351 of the Indian Constitution deals with
the official languages.
• There are 22 scheduled languages in the Eighth Schedule. They are - Assamese,
Bengali, Gujarati, Hindi, Kannada, Kashmiri, Konkani, Malayalam, Manipuri, Marathi,
Nepali, Odia, Punjabi, Sanskrit, Sindhi, Tamil, Telugu, Urdu, Bodo, Santhali, Maithili
and Dogri.
• The Sanskrit language is included in the eighth schedule of the Indian constitution.
Hence statement 1 is correct.
• Viswa Samskrita Dinam is an annual event celebrated on 22nd August. Its objective is
to promote the revival and maintenance of the Sanskrit language. Hence statement
2 is correct.
• It is celebrated for the first time in the year 1969. It is celebrated on Poornima day of
the Shraavana month in the Hindu calendar.
• Sudharma is the only Sanskrit newspaper in the world. It is published since 1970
from Mysore in Karnataka. It is also available in online mode. Hence statement 3 is
correct.
• Mattur, a village in the Shimoga district of Karnataka is believed to have preserved
the Sanskrit language.
Source: https://www.thehindu.com/news/national/andhra-pradesh/without-sanskrit-india-
will-not-be-india-says-kerala-governor/article67072021.ece

Q.6) Solution (d)

Statement Analysis:

• The “Parrot Lady of Khajuraho” is a 900-year-old sandstone sculpture. It is a three-


foot-long sculpture that is exquisitely carved and depicts a lady with a parrot near
her right ear which symbolises love. It was returned to India from Canada. Hence
statement 1 is incorrect.
.
IASBABA’S PRELIMS TEST SERIES (AIPTS) 2024 – CURRENT AFFAIRS (JULY 2023) SOLUTIONS

• It was returned to India according to the UNESCO Convention of 1970. Hence


statement 2 is correct.
• The UNESCO 1970 Convention on the Means of Prohibiting and Preventing the Illicit
Import, Export, and Transfer of Ownership of Cultural Property is an international
treaty to combat the illegal trade in cultural items.
• It was signed on 14 November 1970 and came into effect on 24 April 1972.

Q.7) Solution (a)

Statement Analysis:

• Mata niPachedi is a handmade textile of Gujarat. It was made by members of the


Waghris nomadic tribe. It pays homage to different incarnations of Mata, the divine
singular form of the goddess, and displays narrative depictions of epics of Mata or
Devi or Shakti. It was presented to UK’sPrime Minister. Hence statement 1 is
correct.
• Kanal Brass Set are traditional musical instruments of Himachal Pradesh. It is a
straight brass trumpet now used as decor objects. It was gifted to Spain's Prime
Minister. Hence statement 2 is incorrect.
• Pithora is a ritualistic tribal mural folk art of Gujarat. It is crafted by members of the
Rathwa community. It reflects the social, cultural, and mythological life and beliefs of
these tribals. It was gifted to Australia's Prime Minister. Hence statement 3 is
incorrect.
Pithora Art

Kanal Brass Set


.
IASBABA’S PRELIMS TEST SERIES (AIPTS) 2024 – CURRENT AFFAIRS (JULY 2023) SOLUTIONS

Mata niPachedi
.
IASBABA’S PRELIMS TEST SERIES (AIPTS) 2024 – CURRENT AFFAIRS (JULY 2023) SOLUTIONS

Q.8) Solution (b)

Statement Analysis:

• Sozni also known as Sozan Kaari is a popular needlepoint embroidery technique from
Kashmir valley in Northern India. it has been practiced by Kashmiri artisans since
almost 500 years ago. Hence option b is correct.
• It's mainly done in woolen and silk fabrics.
• It is very famous for its use in Pashmina Cashmere shawls and jackets. Its intricacy
can vary from 5 stitches per cm to 500 stitches per cm.

Q.9) Solution (b)

Statement Analysis:

• A report on donations made to political parties was released by the Association of


Democratic Reforms (ADR). It analyzed donations received from electoral bonds,
direct corporate donations, contributions from MPs/MLAs, meetings, morchas, and
collection by party units.
• It has covered donations received by seven national parties and 24 regional
parties between 2016-17 and 2021-22.
• The donations made to political parties through electoral bonds are higher than by
the corporate sector. The 31 political parties received around total donations of
₹16,437.635 crore out of which 55.90% came from electoral bonds, 28.07% from the
corporate sector, and 16.03% from other sources. Hence statement 1 is incorrect.
• Both national parties and regional parties experienced increased donations from
electoral bonds.
• The donations made to political parties through electoral bonds increased by 743%
between FY 2017-18 and FY 2021-22. Hence statement 2 is correct.

Note: The Association of Democratic Reforms (ADR) is an Indian Non-governmental


Organization (NGO). It was established in 1999 and is situated in New Delhi.
Source: https://www.thehindu.com/news/national/electoral-bonds-chief-source-of-
donations-for-political-parties-report/article67068053.ece#:~:text=SUBSCRIBE-
,Between%202016%2D17%20and%202021%2D22%2C%20the%20seven%20national,by%20
the%20Association%20of%20Democratic

Q.10) Solution (c)

Statement Analysis:

The National Human Right Commission was established under the Protection of Human
Rights Act (PHRA) of 1993.
A few major issues which are taken up by NHRC are:
.
IASBABA’S PRELIMS TEST SERIES (AIPTS) 2024 – CURRENT AFFAIRS (JULY 2023) SOLUTIONS

• Custodial Torture
• Right to Work and Labour Rights
• Arbitrary Arrest and Detention
• Excessive Powers of the Armed Forces and the Police
• Sexual Violence
• Child Labour
• Violence and discrimination against Women, Children
• Lesbian, Gay, Bisexual, and Transgender Rights
Hence option c is correct.
Source: https://www.thehindu.com/news/international/un-rights-body-passes-pakistan-
motion-on-religious-hatred-after-quran-burning-in-sweden-india-votes-in-
favour/article67071945.ece
https://nhrc.nic.in/press-release/important-cases-6

Q.11) Solution (b)

Statement Analysis:

• Siachen Glacier is under the administration of India, post Operation Meghdoot.


Hence statement 1 is incorrect.
• The Indian Armed Forces carried out Operation Meghdoot to seize Siachen Glacier in
Kashmir in 1984.
• Fedchenko Glacier in Tajikistan is the world’s longest glacier in non-polar areas.
• Siachen Glacier is the world’s second longest glacier in non-polar areas. Hence
statement 2 is incorrect.
• It is the world's highest battlefield. It islocated in the Eastern Karakoram range in the
Himalayas. Hence statement 3 is correct.
• It is the place where the Nubra River originates. Hence statement 4 is correct.
.
IASBABA’S PRELIMS TEST SERIES (AIPTS) 2024 – CURRENT AFFAIRS (JULY 2023) SOLUTIONS

Source: https://www.thehindu.com/news/national/the-first-gsi-survey-of-the-
siachen/article67077315.ece#:~:text=In%20June%201958%2C%20V.%20K.,the%20glacier%2
0since%20the%20beginning

Q.12) Solution (d)

Statement Analysis:

• Phosphine (PH3) is a phosphorus atom with three hydrogen atoms attached.


• It can be made naturally and industrially. Hence statement 1 is incorrect.
• It can be manufactured industrially from white phosphorus by hydrolysis with an
alkali metal hydroxide or an aqueous acid in a catalyzed disproportionation reaction.
Hence statement 2 is correct.
• It can be manufactured naturally by some species of anaerobic bacteria.
Source: https://www.livemint.com/science/news/is-there-life-on-venus-heres-what-new-
evidence-of-phosphine-in-clouds-of-venus-indicates-11689040248584.html

Q.13) Solution (b)

Statement Analysis:

Dam River
.
IASBABA’S PRELIMS TEST SERIES (AIPTS) 2024 – CURRENT AFFAIRS (JULY 2023) SOLUTIONS

1. Rana Pratap Sagar Dam Chambal River

2. Tajewala Barrage Dam Yamuna River

3. Rajghat Dam Betwa River

4. Gangau Dam Ken River

• Dams on the Chambal River - Gandhi Sagar Dam, the Jawahar Sagar Dam, and the
Kota Barrage.
• Dams on Yamuna River –Lakhwar-Vyasi Dam
Hence option b is correct.
Source: https://www.thehindu.com/news/cities/Delhi/yamuna-water-level-breaches-60-
year-record-kejriwal-seeks-centres-
help/article67073258.ece#:~:text=With%20the%20Yamuna%20water%20level,highest%20r
ecorded%20level%20since%201963

Q.14) Solution (c)

Statement Analysis:

The Places of Worship Act of 1991 was enacted to prohibit conversion of any place of
worship and to provide for the maintenance of the religious character of any place of
worship as it existed on the 15th of August 1947.
Its major provisions are:
• Section 3 of the Act bars the conversion, in full or part, of a place of worship of any
religious denomination into a place of worship of a different religious denomination,
or even a different segment of the same religious denomination.
• Section 4(1) of the Act declares that the religious character of a place of worship
shall continue to be the same as it existed on August 15, 1947. Hence statement 1 is
correct.
• Section 5 of the Act shall not apply to the Ram Janmabhoomi-Babri Masjid case, and
any suit, appeal, or proceeding relating to it. Hence statement 2 is correct.
• Section 6 of the Act prescribes a punishment of a maximum of three years
imprisonment along with a fine for contravening the provisions of the Act. Hence
statement 3 is correct.
• Section 4(2) of the Act declares any suit or legal proceeding with respect to the
conversion of the religious character of any place of worship existing on August 15,
1947, pending before any court, shall abate — and no fresh suit or legal proceedings
shall be instituted.
Source: https://www.thehindu.com/news/national/validity-of-places-of-worship-act-sc-
gives-centre-time-to-clarify-stand-adjourns-case-to-october-31/article67066721.ece
.
IASBABA’S PRELIMS TEST SERIES (AIPTS) 2024 – CURRENT AFFAIRS (JULY 2023) SOLUTIONS

Q.15) Solution (b)

Statement Analysis:

• Hampi is located in the Tungabhadra basin in Bellary District, Central Karnataka.


Hence statement 1 is correct.
• It was the capital city during Sangama, Saluva, Tuluva, and Aravidu.
• Thanjavur (Tanjore) was the capital of Cholas. Hence statement 2 is incorrect.
• It is a UNESCO World Heritage Site since 1986 and is called the “World’s Largest
Open-air Museum”. Hence statement 3 is correct.
• Hampi is the world’s second-largest medieval-era city after Beijing.

Places in Hampi

Source: https://newsonair.gov.in/News?title=3rd-Sherpas-G20-meeting-to-begin-in-
Hampi%2c-Karnataka&id=464139

Q.16) Solution (b)

Statement Analysis:

• SAGAR SAMPARK is an indigenous Differential Global Navigation Satellite System


(DGNSS) launched by the Ministry of Ports, Shipping, and Waterways. Hence
statement 1 is incorrect.
• DGNSS is a terrestrial-based enhancement system that corrects the errors and
inaccuracies in the Global Navigation Satellite System (GNSS) allowing for more
accurate positioning information.
.
IASBABA’S PRELIMS TEST SERIES (AIPTS) 2024 – CURRENT AFFAIRS (JULY 2023) SOLUTIONS

• It provides safe navigation, reducing the risk of collisions, groundings, and accidents
at sea. Hence statement 2 is correct.
• It will lead to the safe and efficient movement of vessels.
• It improves the accuracy of GPS positioning by reducing errors caused byatmospheric
inferences, satellite clock drift, and other factors.
Source:https://pib.gov.in/PressReleaseIframePage.aspx?PRID=1938985

Q.17) Solution (d)

Statement Analysis:

Sariska Tiger Reserve is nestled in the Aravali hills in Rajasthan with mountains, grasslands,
dry deciduous forests, and cliffs. It was declared a wildlife sanctuary in 1955 and a tiger
reserve in 1978. It has a blend of tropical dry deciduous and tropical thorn forests.
Kankarwadi Fort is located in the center of the reserve. It houses the Lord Hanuman temple
and Neelkanth temple. Hence option d is correct.
.
IASBABA’S PRELIMS TEST SERIES (AIPTS) 2024 – CURRENT AFFAIRS (JULY 2023) SOLUTIONS

Source: https://www.hindustantimes.com/cities/jaipur-news/tigress-st-19-gives-birth-to-
two-cubs-increasing-tiger-population-in-sariska-tiger-reserve-to-30-101688899410204.html

Q.18) Solution (b)

Statement Analysis:

• Perseverance Rover’s primary objective is to look for signs of ancient microbial life
on Marsand to study and analyze the Red Planet’s regolith, rock, and dust. Hence
statement 1 is incorrect.
.
IASBABA’S PRELIMS TEST SERIES (AIPTS) 2024 – CURRENT AFFAIRS (JULY 2023) SOLUTIONS

• It is the first rover to collect and cache samples.


• It will manufacture molecular oxygen on Mars using MOXIE. It produces oxygen using
carbon dioxide from the carbon dioxide-rich atmosphere. Hence statement 2 is
correct.
• It has the first-ever helicopter to fly on Mars called Ingenuity. This is the first time
NASA will fly a helicopter on another planet or satellite. Hence statement 3 is
correct.
• A Multi-Mission Radioisotope Thermoelectric Generator (MMRTG) on the rover is its
power source. It converts heat from the natural radioactive decay of plutonium
(Plutonium Dioxide) into electricity.

Source: https://www.downtoearth.org.in/news/science-technology/life-beyond-earth-nasa-
s-perseverance-rover-finds-diverse-organic-matter-on-mars-90595

Q.19) Solution (b)

The GrossNon-Performing Assets (GNPA) of the country’s scheduled commercial banks,


which declined to a 10-year low of 3.9 per cent in March 2023, is expected to fall further to
3.6 per cent by March 2024, the Reserve Bank of India (RBI).

If the macroeconomic environment worsens, though, the GNPA ratio may rise sharply, the
central bank said in its Financial Stability Report (FSR) for June 2023..

Q.20) Solution (a)

Statement Analysis:

• Section 144 of the Code of Criminal Procedure (CrPC) of 1973 places restrictions on
handling or transporting any kind of weapon in the given jurisdiction. Hence
statement 1 is correct.
• According to the order under this section, there shall be no movement of the public
and all educational institutions shall remain closed. There will be a complete bar on
.
IASBABA’S PRELIMS TEST SERIES (AIPTS) 2024 – CURRENT AFFAIRS (JULY 2023) SOLUTIONS

holding any kind of public meeting or rally during the period of operation of this
order.
• It empowers a district magistrate, a sub-divisional magistrate or any other executive
magistrate specially empowered by the state government in this behalf to issue
orders to prevent and address urgent cases of apprehended danger or nuisance.
Hence statement 2 is correct.
• The order under this section cannot remain in force for a period of more than two
months. Under the state’s discretion power, the order can be extended for two more
months with a maximum validity extendable to six months. Hence statement 3 is
incorrect.

Source: https://indianexpress.com/article/explained/everyday-explainers/delhi-police-
prohibitory-orders-section-144-flood-8831180/

Q.21) Solution (b)

Statement Analysis:

• The National Maritime Heritage Complex is being constructed by the Ministry of


Ports, Shipping, and Waterways. It aims to showcase the maritime heritage of India
from ancient to modern times. Hence statement 1 is correct.
• It is being constructed in the Indus Valley civilization region of Lothal, Gujarat. Hence
statement 2 is incorrect.
• It will have several innovative features such as Lothal mini-recreation, which will
recreate Harappan architecture and lifestyle through immersive technology; four
theme parks – Memorial theme park, Maritime and Navy theme park, Climate theme
park, and Adventure and Amusement theme park.
• It will house Asia’s biggest underwater marine museum, India’s grandest naval
museum, and the world’s tallest lighthouse museum. Hence statement 3 is correct.
.
IASBABA’S PRELIMS TEST SERIES (AIPTS) 2024 – CURRENT AFFAIRS (JULY 2023) SOLUTIONS

Source: https://www.financialexpress.com/business/infrastructure-gujarat-set-to-house-
national-maritime-heritage-complex-showcasing-indias-rich-maritime-legacy-check-its-
features-and-more-3151170/

Q.22) Solution (b)

Statement Analysis:

• 125th Birth Anniversary of Alluri Sitarama Raju.


• Alluri Sitarama Raju is considered as Manyam Veerudu for his bravery and sacrifice.
Hence statement 1 is correct.
• He led a guerrilla campaign in the Eastern Ghats region of Andhra Pradesh. Hence
statement 2 is correct.
• He opposed the British in response to the Madras Forest Act of 1882 in the backdrop
of the Non-cooperation movement. Hence statement 3 is incorrect.
.
IASBABA’S PRELIMS TEST SERIES (AIPTS) 2024 – CURRENT AFFAIRS (JULY 2023) SOLUTIONS

• The Madras Forest Act of 1882 effectively restricted the free movement
of Adivasis in their forest habitats and prevented them from practicing their
traditional form of agriculture called 'podu', which threatened their very way of life.
It led to the Rampa rebellion (1922-1924) in which Alluri Sitarama Raju played the
major role as its leader.

Source:https://pib.gov.in/PressReleasePage.aspx?PRID=1937352

Q.23) Solution (c)

Statement Analysis:

• India Mobile Congress (IMC) is the largest telecom, media, and technology forum in
Asia. It is a leading forum for bringing together industry, government, academia, and
other ecosystem players.
• The theme of the India Mobile Congress-2023 is “Global Digital Innovation”. Hence
statement 1 is correct.
• The India Mobile Congress is jointly organized by the Department of
Telecommunications (DoT) and the Cellular Operators Association of India (COAI).
Hence statement 2 is correct.
• The sub-themes of the India Mobile Congress-2023 is “Engage, Learn, Innovate”.
Source: https://newsonair.gov.in/News?title=New-Delhi%3a-Seventh-edition-of-
Asia%e2%80%99s-premier-digital-technology-exhibition%2c-IMC-to-be-held-from-27th-
October-at-Pragati-Maidan&id=464128

Q.24) Solution (b)

Statement Analysis:

• National Bank for Agriculture and Rural Development (NABARD) is a statutory body
formed in 1982 under the National Bank for Agriculture and Rural Development Act
of 1981. Hence statement 1 is incorrect.
.
IASBABA’S PRELIMS TEST SERIES (AIPTS) 2024 – CURRENT AFFAIRS (JULY 2023) SOLUTIONS

• It isfully owned by the Government of India and is headquartered in Mumbai.


• It deals with the operations of giving credit for agriculture and other economic
activities in rural areas. Hence statement 2 is correct.
• It is a refinancing agency for those institutions that provide investment and
production credit for promoting several developmental programs for rural
development.
Source: https://newsonair.gov.in/News?title=NABARD-is-ensuring-development-and-
prosperity-in-rural-areas-by-making-rural-banking-accessible%2c-says-Home-Minister-Amit-
Shah&id=464117

Q.25) Solution (b)

Statement Analysis:

The Solomon Islands is found in the southwestern Pacific Ocean with Honiara as its capital.
It lies between Papua New Guinea and Vanuatu. It is a sovereign country consisting of six
major islands and over 900 smaller islands in Oceania. Hence option b is correct.

Source: https://www.thehindu.com/news/international/solomon-islands-leader-visits-
beijing-highlighting-us-china-rivalry-in-south-pacific/article67064535.ece

Q.26) Solution (a)

Statement Analysis:

• Senior Citizen Savings Scheme provides a regular income to senior citizens of India
after they attain the age of 60 years. Hence statement 1 is incorrect.
• Indian citizens above the age of 60 years, retirees in the age bracket of 55-60
years who have opted for a Voluntary Retirement Scheme (VRS) or Superannuation,
and retired defence personnel above 50 years and below 60 years of age are eligible
for this scheme.
.
IASBABA’S PRELIMS TEST SERIES (AIPTS) 2024 – CURRENT AFFAIRS (JULY 2023) SOLUTIONS

• Hindu Undivided Family (HUFs) and Non-resident Indians (NRIs) are not eligible for
this scheme. Hence statement 2 is correct.
• Individuals are allowed to operate more than one account by themselves or open a
joint account with their spouse.
• The eligible investors can make a minimum deposit of Rs. 1000 and a maximum
deposit of Rs. 30 lakhs. Hence statement 3 is incorrect.
• It has a maturity period of five years. But, a depositor can extend one’s maturity
period for another three years.
• It qualifies for deduction under section 80-C of the Income Tax Act.

Note: Section 80C of the Income Tax Act allows to reduce taxable income by making tax-
saving investments or incurring eligible expenses.
Source: https://www.businesstoday.in/bt-tv/video/the-senior-citizen-savings-scheme-
boost-82-interest-among-the-highest-389374-2023-07-12

Q.27) Solution (b)

Statement Analysis: Both the statements are individually correct.

• SSLV is an indigenously developed mini rocket launcher.It is the smallest vehicle at


ISRO with a 110-ton mass.
• It is specially designed to carry smaller commercial satellites into the low-earth orbit
(LEO) from 200-2,000 km above the Earth’s surface.
• It is a three-stage Launch Vehicle configured with three Solid Propulsion Stages and a
liquid propulsion-based Velocity Trimming Module (VTM) as a terminal stage.Hence
statement 2 is correct.
• The Small Satellite Launch Vehicle (SSLV) is more economical than other satellite
launch vehicles of ISRO.
• SSLV will take only 72 hours to integrate, unlike the 70 days taken now for other
launch vehicles like PSLV.
• SSLV has minimal launch infrastructure requirements.
• SSLV requires less labour force as only six people will be required to do the job,
instead of 60 people for other launch vehicles. Hence statement 1 is correct.
• The SSLV can deliver 600 kg to Low Earth Orbit (500 km) and 300 kg to Sun-
synchronous Orbit (500 km).
.
IASBABA’S PRELIMS TEST SERIES (AIPTS) 2024 – CURRENT AFFAIRS (JULY 2023) SOLUTIONS

IMAGE SOURCE: Bharat Rakshak


Note: Low-earth orbit (LEO) is an orbit that is relatively close to Earth’s surface.It is normally
at an altitude of less than 1000 km but could be as low as 160 km above Earth.
Source: https://newsonair.gov.in/News?title=IN-SPACe-calls-for-Expression-of-Interest-
from-Indian-private-Industries-to-manufacture-SSLV&id=464064

Q.28) Solution (c)

Statement Analysis:

• Non-Fungible Tokens (NFTs) are assets in the digital world that can be bought and
sold like any other piece of property. Hence statement 1 is correct.
• They can be tweets, drawings, photos, videos, GIFs, music, in-game items, and
selfies. Hence statement 2 is correct.
• They are backed by blockchain technology and can have only one owner at a time.
Hence statement 3 is incorrect.
• NFT owners can also digitally sign their artwork and store specific information in
their NFTs' metadata. This will be only viewable to the individual who bought the
NFT.
• They can be bought by anyone who holds a cryptocurrency wallet. Hence statement
4 is correct.
Source: https://newsonair.gov.in/News?title=Home-Minister-Amit-Shah-to-address-two-
days-G20-Conference-on-Crime-and-Security-in-Age-of-NFTs%2c-AI-on-13th-
July&id=464052

Q.29) Solution (b)

Statement Analysis:

• Brucellosis is a bacterial zoonotic disease. Hence statement 1 is incorrect.


• Its symptoms include fever, weakness, malaise, and weight loss.
.
IASBABA’S PRELIMS TEST SERIES (AIPTS) 2024 – CURRENT AFFAIRS (JULY 2023) SOLUTIONS

• It is also known as Malta fever or Mediterranean fever.


• It is caused by various Brucella species, which mainly infect cattle, swine, goats,
sheep, and dogs.
• Brucella melitensis is the most prevalent species causing human brucellosis.
• It affects people of all ages and both sexes. Hence statement 2 is correct.
• It can be caused by eating raw or unpasteurized dairy products.
• It can spread through the air or through direct contact with infected animals. Hence
statement 3 is correct.
• It can usually be treated with antibiotics. However, treatment takes several weeks to
months, and the infection can recur.
Source: https://www.newindianexpress.com/states/kerala/2023/jul/12/girl-in-kollam-
infected-with-brucellosis-2593827.html

Q.30) Solution (a)

Statement Analysis:

• Candida auris is a multi-drug-resistant fungus that can cause invasive infections in


the human body. Hence statement 1 is correct.
• It can spread indirectly from patient to patient in healthcare settings such as
hospitals or nursing homes.
• It mainly affects patients who already have many medical problems or have had
frequent hospital stays or live in nursing homes.
• Its symptoms include fever and chills that do not go away after treatment with
antibiotics.
• It is treated with an antifungal drug named echinocandins.
• Adefovir is an antiviral drug used to treat Hepatitis B. Hence statement 2 is
incorrect.
Source: https://www.thehindu.com/news/national/drug-resistant-fungus-identified-as-a-
global-threat-found-in-hospitalised-stray-dogs-in-delhi/article67058034.ece

Q.31) Solution (b)

Statement Analysis:

• The Directorate of Revenue Intelligence (DRI) is the apex anti-smuggling agency of


India. Hence statement 1 is correct.
• It works under the Department of Revenue, Ministry of Finance, Government of
India. Hence statement 2 is incorrect.
• DRI enforces the provisions of the Customs Act, Arms Act, NDPS Act, COFEPOSA,
Wildlife Act, Antiquities Act, etc. Hence statement 3 is correct.
.
IASBABA’S PRELIMS TEST SERIES (AIPTS) 2024 – CURRENT AFFAIRS (JULY 2023) SOLUTIONS

• DRI undertakes the collection, collation, analysis, and dissemination of intelligence


relating to smuggling, carries out investigations, adjudication of cases, and
prosecution of the arrested persons.
Source: https://newsonair.gov.in/News?title=DRI-seize-48-kg-of-gold-paste-worth-25-crore-
rupees-at-Surat-international-airport-from-three-passengers-travelling-from-
Sharjah&id=463959

Q.32) Solution (c)

Statement Analysis:

• Unified Payments Interface, widely known as UPI across India, is developed by the
National Payments Corporation of India (NPCI).
• UPI serves as an amalgamation of several banking features to ensure ideal merchant
payment and routing of funds. In the form of a single mobile application, UPI enables
instant money transfers, access to different bank accounts, bill payments, and Quick
Response (QR) code payments, to name a few. Hence option c is correct.
Source: https://www.thehindubusinessline.com/topic/UPI/

Q.33) Solution (d)

Statement Analysis:

• Hansa Jivraj Mehta participated in the Non-Cooperation and Swadeshi movements.


Her involvement with India’s freedom struggle got her arrested in 1932. Hence
statement 1 is correct.
• She was a member of the Bombay Legislative Council in 1937. She took on the
elections as a general category candidate and remained in the council until 1949.
Hence statement 2 is correct.
• She became the president of the All India Women’s Conference in 1946. During her
presidency, she drafted the Indian Women’s Charter of Rights and Duties, which
demanded gender equality and civil rights for women.
• Mehta was one of the 15 women framers of the Indian Constitution who was
elected to the Constituent Assembly from Bombay. She was a member of the
constituent assembly who favoured a uniform civil code and women’s rights. Hence
statement 3 is correct.
• In 1946, Mehta served as a member of the United Nations sub-committee on the
status of women. She was the vice-chair with Eleanor Roosevelt of the United
Nations Universal Declaration of Human Rights Committee. She played a major role
in ensuring marriage equality for women through Article 16 of the Universal
Declaration of Human Rights (UDHR). Hence statement 4 is correct.
• She became the first female Vice-Chancellor in India with her appointment at the
Maharaja SNDT University in Bombay.
.
IASBABA’S PRELIMS TEST SERIES (AIPTS) 2024 – CURRENT AFFAIRS (JULY 2023) SOLUTIONS

• The Indian government awarded her with the Padma Bhushan in 1959.
https://english.newstracklive.com/news/remembering-hansa-jivraj-mehta-an-inspirational-
indian-reformist-and-feminist-sc1-nu318-ta318-1282541-1.html

Q.34) Solution (a)

Statement Analysis:

• The Kisan drones consist of unmanned tanks filled with insecticides and nutrients
with a capacity of 5 to 10kg.
• Kisan drones will save the time of the farmers and do work with less effort. Hence
statement 1 is correct.
• Kisan drones can spray uniformly the same amount of pesticide in less time and can
carry vegetables and fruits from farm to market with minimal damage. Hence
statement 2 is correct.

Note: The Drone Shakti initiative aimed to promote and facilitate drones as a service
through start-ups. It is aimed to increase domestic manufacturing of drones and create
employment.
Source: https://www.thehindu.com/news/cities/puducherry/kisan-drone-to-be-ready-for-
commercial-use-soon-it-will-help-in-various-farm-operations/article66915863.ece

Q.35) Solution (b)

Statement Analysis:

• Performance Grading Index for Districts (PGI-D) was released by the Department of
School Education and Literacy, Ministry of Education. Hence statement 1 is correct.
• Its objective is to help the districts to prioritize areas for intervention in school
education and improve to reach the highest grade. Hence statement 2 is correct.
• Daksh is the highest grade in PGI-D whereas Akanshi-3 is the lowest grade in PGI-D.
Hence statement 3 is incorrect.
Source: https://newsonair.gov.in/News?title=Education-Ministry-releases-Performance-
Grading-Index-for-Districts-combined-report-for-2020-21-%26--2021-22&id=463936

Q.36) Solution (a)

Statement Analysis:

• Nitrogen Dioxide (NO2) is an odourless, acidic, and extremely corrosive gas. It is


a highly reactive gas. Hence statement 1 is correct.
• It causes air pollution, water pollution, and acid rain. Hence statement 2 is correct.
.
IASBABA’S PRELIMS TEST SERIES (AIPTS) 2024 – CURRENT AFFAIRS (JULY 2023) SOLUTIONS

• It occurs naturally and industrially. It forms from emissions from cars, trucks and
buses, power plants, and off-road equipment. It is created by natural sources that
include volcanoes and microbes. Hence statement 3 is incorrect.
Source: https://www.downtoearth.org.in/news/pollution/no2-exposure-can-lead-to-
premature-death-respiratory-circulatory-illness-report-90457

Q.37) Solution (a)

Statement Analysis:

• Indian Grey Hornbill are arboreal and found in the foothills of the Southern
Himalayas.
• They are recently spotted in Puducherry for the first time and previously near the
Sankarabarani River in Tamil Nadu.Hence statement 1 is incorrect.
• They are listed as least concern on the IUCN List. Hence statement 2 is incorrect.
• They are the smallest in the Hornbill family. Hence statement 3 is correct.
Source: https://www.thehindu.com/news/cities/puducherry/first-recorded-sighting-of-
indian-grey-hornbill-in-puducherry/article67052557.ece

Q.38) Solution (b)

Statement Analysis:

• SEBI’s Complaints Redress System (SCORES ) platform is launched by the Securities


and Exchange Board of India (SEBI). Hence statement 1 is incorrect.
• It does not deal with complaints against companies. It covers complaints related
to investment advisers, research analysts, corporate governance/listing conditions,
minimum public shareholding, venture capital funds, and takeover/restructuring.
Hence statement 2 is correct.
• It deals with complaints under the Companies Act of 2013, the SEBI Act, the
Securities Contract Regulation Act, and the Depositories Act. Hence statement 3 is
correct.
• The complaint shall be lodged on SCORES within one year from the date of the cause
of action and the average resolution time for a complaint is 31 days.
Source: https://www.business-standard.com/markets/news/sebi-s-scores-platform-
disposes-of-3-079-complaints-in-june-data-123070800658_1.html

Q.39) Solution (c)

Statement Analysis:

• LCA Tejas is an indigenous, lightweight, single-engine, delta wing, multipurpose light


fighter, for the Indian Air Force and Indian Navy.
.
IASBABA’S PRELIMS TEST SERIES (AIPTS) 2024 – CURRENT AFFAIRS (JULY 2023) SOLUTIONS

• It is developed in both fighter and trainer versions. It is a multipurpose supersonic


aircraft. Hence statement 1 is correct.
• It can carry a payload of up to 4000 kg with a top speed of Mach 1.8.
• It is manufactured by Hindustan Aeronautics Limited (HAL) and is designed by
Aeronautical Development Agency. Hence statement 2 is correct.
Source: https://hal-india.co.in/Product_Details.aspx?Mkey=54&lKey=&CKey=20

Q.40) Solution (d)

Statement Analysis:

Dinutuximab (Qarziba) is a cancer medicine used to treat neuroblastoma, a cancer of nerve


cells, in patients over 1 year of age.Hence option d is correct.
Source: https://indianexpress.com/article/business/gst-council-exempt-igst-cancer-drug-
import-tax-8790397/

Q.41) Solution (b)

Statement Analysis:

• Ubinas is an active stratovolcano located in the Moquegua Region of southern Peru,


approximately 60 kilometers east of the city of Arequipa.
• It is part of the Central Volcanic Zone of the Andes and stands at an elevation of
5,672 meters above sea level. Hence option b is correct.

IMAGE SOURCE: researchgate.net


Source: https://newsonair.gov.in/News?title=Peru-declares-state-of-emergency-for-60-
days-in-areas-around-Ubinas-volcano&id=463767

Q.42) Solution (d)


.
IASBABA’S PRELIMS TEST SERIES (AIPTS) 2024 – CURRENT AFFAIRS (JULY 2023) SOLUTIONS

Statement Analysis:

Gramodyog Vikas Yojana (GVY) scheme is for the promotion and development of village
industries through common facilities, technological modernization, training, etc., and other
support and services for the promotion of village Industries.
GVY has the following components/ verticals from the activities under Village Industries:
• Wellness & Cosmetics Industry (WCI)
• Handmade Paper, Leather, & Plastic Industry (HPLPI)
• Agro-Based & Food Processing Industry (ABFPI)
• Mineral-Based Industry (MBI)
• Rural Engineering & New Technology Industry (RENTI)
• Service Industry
Hence option d is correct.
Source:https://msme.gov.in/sites/default/files/RevisedoperationalGuidelinesofGVY.pdf

Q.43) Solution (c)

Statement Analysis:

The World Investment Report 2023 is published by the United Nations Conference on Trade
and Development (UNCTAD). It focuses on trends in Foreign Direct Investment (FDI)
worldwide, at the regional and country levels, and emerging measures to improve its
contribution to development.
The reports published by the United Nations Conference on Trade and Development
(UNCTAD) are:
• Trade and Development Report
• World Investment Report
• The Least Developed Countries Report
• Information and Economy Report
• Technology and Innovation Report
• Commodities and Development Report
Hence option c is correct.
Source:https://www.downtoearth.org.in/news/world/ukraine-high-food-and-energy-prices-
debt-global-fdi-fell-12-in-2022-due-to-multiple-crises-90425

Q.44) Solution (b)

Statement Analysis:

According to the World Investment Report 2023:


• India and ASEAN registered an increase in foreign direct investment (FDI). Hence
statement 1 is incorrect.
.
IASBABA’S PRELIMS TEST SERIES (AIPTS) 2024 – CURRENT AFFAIRS (JULY 2023) SOLUTIONS

• China is the second-largest FDI host country in the world. Hence statement 2 is
correct.
• FDI registered a decline in Gulf countries and the least developed countries. Hence
statement 3 is correct.
• FDI inflows were higher in developing countries.
• Much of the growth in international investment in renewable energy has been
concentrated in developed countries.
• The investment gap across all sectors of the Sustainable Development Goals
has increased to more than $4 trillion per year from $2.5 trillion in 2015. The largest
gaps are in energy, water, and transport infrastructure.
Source: https://www.downtoearth.org.in/news/world/ukraine-high-food-and-energy-
prices-debt-global-fdi-fell-12-in-2022-due-to-multiple-crises-90425

Q.45) Solution (c)

Statement Analysis:

• Similipal National Park is a national park and a tiger reserve located in Odisha. Hence
statement 1 is incorrect.
• It is part of the Similipal-Kuldiha-Hadgarh Elephant Reserve popularly known as the
Mayurbhanj Elephant Reserve.
• It is a part of the UNESCO World Network of Biosphere Reserves.
• It has beautiful waterfalls like Joranda and Barehipani.
• It is surrounded by the twin peaks of Khairiburu and Meghashini. Hence statement 2
is correct.
• Rivers like Burhabalanga, Palpala Bandan, Salandi, Kahairi, and Deo flow through it.
• Its vegetation is a mix of deciduous and semi-evergreen forests. Hence statement 3
is correct.
• It houses tribes like Kolha, Santhala, Bhumija, Bhatudi, Gondas, Khadia, Mankadia,
and Sahara. Hence statement 4 is correct.
Source: https://www.downtoearth.org.in/news/wildlife-biodiversity/over-100-armed-
police-deployed-in-similipal-national-park-as-forest-staff-go-on-strike-90414

Q.46) Solution (a)

Statement Analysis:

• James Webb Space Telescope is an infrared observatory orbiting the Sun.It was
launched in 2021 on an Ariane 5 rocket from French Guiana. Hence statement 1 is
incorrect.
• It is launched by the National Aeronautics and Space Administration (NASA). It is
NASA’s largest and most powerful space science telescope. Hence statement 2 is
incorrect.
.
IASBABA’S PRELIMS TEST SERIES (AIPTS) 2024 – CURRENT AFFAIRS (JULY 2023) SOLUTIONS

• Its objective is to find the first galaxies that formed in the early universe and to see
stars forming planetary systems.
• It aims to study every phase in the history of our universe, ranging from the first
luminous glows after the big bang, to the formation of solar systems capable of
supporting life on planets like Earth, to the evolution of our solar system. Hence
statement 3 is correct.

IMAGE SOURCE: jpl.nasa.gov

Source: https://indianexpress.com/article/explained/explained-sci-tech/detecting-carbon-
molecules-space-jwst-8699417/

Q.47) Solution (b)

Statement Analysis:

• Special Window for Funding Stalled Affordable and Middle-Income Housing Projects
(SWAMIH) Fund is a social impact fund specifically formed for completing stressed
and stalled residential projects. Hence statement 1 is correct.
• It is available only for projects that fall under the affordable and mid-income housing
categories.
• To be eligible for funding projects must be registered under the Real Estate
(Regulation and Development) Act (RERA) 2016, or projects must be classified as a
non-performing asset (NPA), or be under insolvency proceedings or projects should
have been declared as stalled or delayed project by a competent authority. Hence
statement 2 is correct.
• It is sponsored by the Ministry of Finance, Government of India. Hence statement 3
is incorrect.
Source: https://pib.gov.in/PressReleasePage.aspx?PRID=1937133

Q.48) Solution (c)


.
IASBABA’S PRELIMS TEST SERIES (AIPTS) 2024 – CURRENT AFFAIRS (JULY 2023) SOLUTIONS

Statement Analysis:

• The One Health Priority Research Agenda on Antimicrobial Resistance is launched by


the United Nations (UN) Food and Agriculture Organization (FAO), the UN
Environment Programme (UNEP), World Health Organization (WHO), and the World
Organisation for Animal Health (WOAH). Hence statement 1 is correct.
• Its objective is to prioritise 40 research topics for evidence generation to inform
policy and interventions by 2030 and to guide a variety of stakeholders in generating
new evidence to address antimicrobial resistance(AMR), with a focus on low- and
middle-income countries.
• It is based on pillars such as transmission, integrated surveillance, interventions,
behavioural insights, and change, economics, and policy. Hence statement 2 is
correct.

IMAGE SOURCE: DOWN TO EARTH

Source: https://www.downtoearth.org.in/news/health/one-health-fao-unep-who-and-
woah-launch-research-agenda-for-antimicrobial-resistance-90388

Q.49) Solution (d)

Statement Analysis:

• Gucchi is a species of fungus from the family Morchellaceae of the Ascomycota.


Hence statement 1 is correct.
• It is one of the costliest mushrooms in the world and is known for its spongy,
honeycomb head and savoryflavor.
• They are pale yellow in colour with large pits and ridges on the surface of the cap.
• It is found in temperate forests. It is primarily obtained from the forests and pastures
of Doda district, Jammu and Kashmir. Hence statement 2 is correct.
• It is rich in antioxidant and antimicrobial properties that prevent health issues
including heart diseases and diabetes by removing reactive oxygen species that harm
the body. Hence statement 3 is correct.
.
IASBABA’S PRELIMS TEST SERIES (AIPTS) 2024 – CURRENT AFFAIRS (JULY 2023) SOLUTIONS

Source: https://www.downtoearth.org.in/news/climate-change/impact-of-climate-change-
on-kashmir-s-mushroom-pickers-90344

Q.50) Solution (b)

Statement Analysis:

Wildlife Crime Control Bureau (WCCB) is a statutory body established by amending the
Wildlife Protection Act of 1972 in 2007. Hence statement 1 is incorrect.
Its objective is to combat organized wildlife crime in the country. Hence statement 2 is
correct.
The functions of WCCB include:
• To collect and collate intelligence related to organized wildlife crime activities.
• To disseminate the same to State and other enforcement agencies for immediate
action to apprehend the criminals.
• To establish a centralized wildlife crime data bank;
• Coordinate actions by various agencies in connection with the enforcement of the
provisions of the Act.
• To assist foreign authorities and international organizations concerned to facilitate
coordination and universal action for wildlife crime control.
• Capacity building of the wildlife crime enforcement agencies for a scientific and
professional investigation into wildlife crimes and assist State Governments.
• To ensure success in prosecutions related to wildlife crimes.
Source: https://www.hindustantimes.com/india-news/red-alert-tiger-poaching-rises-as-
wildlife-crime-control-bureau-urges-intensified-patrolling-in-tiger-reserves-
101688284176992.html

Q.51) Solution (a)

Statement Analysis:

• Global Gibbon Network is established by the Hainan Institute of National


Parks and Eco Foundation Global. It aims to protect and preserve Asia’s unique
heritage, specifically the singing Gibbon and their habitats, by promoting
participatory conservation policies, legislation, and actions. Hence statement 1 is
incorrect.
• Hoolock Gibbon is the only ape found in India and is the state animal of Arunachal
Pradesh. They are found in tropical and subtropical forests in the southeastern part
of Asia. Hence statement 2 is incorrect.
• Both Western Hoolock Gibbon and Eastern Hoolock Gibbon are protected under
Schedule I of the Wildlife Protection Act of 1972. Hence statement 3 is correct.
.
IASBABA’S PRELIMS TEST SERIES (AIPTS) 2024 – CURRENT AFFAIRS (JULY 2023) SOLUTIONS

• Western Hoolock Gibbons are found in all the states of the northeast, restricted
between the south of the Brahmaputra River and east of the Dibang River. And
outside India, it is found in eastern Bangladesh and north-west Myanmar. It is listed
as endangered on the IUCN Red List.
• Eastern Hoolock Gibbons are found in specific pockets of Arunachal Pradesh and
Assam in India and southern China and northeast
Myanmar outside India. It is listed as vulnerable on
the IUCN Red List.

Note: International Gibbon Day is celebrated


on October 24 every year. It originated in 2015
when the IUCN Primate Specialist Group established
the day as a commemoration to draw public
attention to gibbon protection.

Source: https://www.thehindu.com/sci-tech/energy-and-environment/china-event-raises-
concern-over-indias-only-ape/article67087123.ece

Q.52) Solution (d)

Statement Analysis:

• Purana cheeveda commonly found in several parts of South India is an insect. They
belong to the cicada species. Hence option d is correct.
• Cicada species belong to the order Hemiptera and the superfamily Cicadoidea. They
have large eyes, transparent wings, and loud calls that are produced by special
organs called tymbals. They are found on every continent except Antarctica. They
are canopy dwellers and are found in natural forests with large trees. They are
herbivorous and feed on plant sap using their piercing and sucking mouthparts.

Source: https://epaper.thehindu.com/ccidist-
ws/th/th_delhi/issues/43910/OPS/G11BG9OHU
.1.png?cropFromPage=true

Q.53) Solution (c)

Statement Analysis:
.
IASBABA’S PRELIMS TEST SERIES (AIPTS) 2024 – CURRENT AFFAIRS (JULY 2023) SOLUTIONS

The strategic roadmap “Horizon 2045” is between India and France. It includes cooperation
in defence, space, nuclear energy, climate change, and green transitions as well as
education and people-to-people ties for the next 25 years. Hence option c is correct.

Source:https://epaper.thehindu.com/ccidist-
ws/th/th_delhi/issues/43910/OPS/G3KBGAFLE.1.png?cropFromPage=true

Q.54) Solution (a)

Statement Analysis:

• PM Mega Integrated Textile Regions and Apparel (PM MITRA) Parks will help in
creating world-class industrial infrastructure by encouraging investment, innovation,
and job creation in the textile sector. Hence statement 1 is correct.
• They are a major step forward in realising the Government's vision of making India a
global hub for textile manufacturing and exports. They will enhance the
competitiveness of the textiles industry by helping it achieve economies of scale as
well as attract global players to manufacture in India.
• It will be implemented by a special purpose vehicle (SPV) owned by the centre and
state through the Ministry of Textiles. Hence statement 2 is incorrect.
• The Ministry of Textiles will provide financial support in the form of Development
Capital Support up to Rs. 500 crore per park to the Park SPV.
• A Competitive Incentive Support (CIS) of up to Rs 300 crore per park to the units in
PM MITRA Park shall also be provided to incentivise speedy implementation.
• State governments will provide contiguous and encumbrance-free land, provision of
all utilities, reliable power supply, water availability, wastewater disposal system, an
effective single window clearance as well as a conducive and stable industrial/textile
policy.

Source: https://pib.gov.in/PressReleseDetail.aspx?PRID=1939995

Q.55) Solution (a)

Statement Analysis:

• A heat wave is defined as a period of abnormally high temperatures over a region.


• In plains, if the temperature reaches at least 40⁰ Celcius, and in hilly regions if the
temperature reaches at least 30⁰ Celcius then it is considered a heat wave.
• Urbanization and concrete structures in cities cause heat waves. Hence statement 1
is correct.
• It is because concrete and asphalt in cities absorb and retain heat leading to
increased temperatures contributing to heat waves. Hence statement 2 is correct
and correct explanation for statement 1.
.
IASBABA’S PRELIMS TEST SERIES (AIPTS) 2024 – CURRENT AFFAIRS (JULY 2023) SOLUTIONS

• An urban heat island is a temporary phenomenon where certain parts of the city
experience a higher temperature than its surroundings. It is due to the heat
trapped in the buildings of cities.

Source: https://epaper.thehindu.com/ccidist-
ws/th/th_delhi/issues/43910/OPS/G3KBGAFGB.1.png?cropFromPage=true

Q.56) Solution (b)

Statement Analysis:

• The UK has signed a deal to join a trade pact with several countries in Asia and the
Pacific, including Japan and Australia. The name of this pact – CPTPP.
• The Comprehensive and Progressive Agreement for Trans-Pacific Partnership
(CPTPP) is a free trade agreement that succeeded the Trans-Pacific Partnership
(TPP).
• TPP was a trade agreement formed in 2005 that consists of 12 member countries.
Later the United States withdrew in 2017 and the grouping came to be called CPTPP.
Hence statement 1 is correct.
• All members of the CPTPP are members of the Asia-Pacific Economic Cooperation
(APEC).
• Members of the CPTPP are Australia, Brunei, Canada, Chile, Japan, Malaysia,
Mexico, New Zealand, Peru, Singapore, and Vietnam.
• Members of APEC are Australia, Brunei Darussalam, Canada, Chile, the People's
Republic of China, Hong Kong, Indonesia, Japan, the Republic of Korea, Malaysia,
Mexico, New Zealand, Papua New Guinea, Peru, the Philippines, the Russian
Federation, Singapore, Chinese Taipei, Thailand, the United States of America, and
Vietnam. Hence statement 2 is incorrect.
• The United Kingdom is the first European country and the first new member of the
bloc since its inception. Hence statement 3 is correct.

Source: https://epaper.thehindu.com/ccidist-
ws/th/th_delhi/issues/43910/OPS/G3KBGAFG9.1.png?cropFromPage=true

Q.57) Solution (c)

Statement Analysis:

• Bihar, Jharkhand, Meghalaya, Uttar Pradesh, and Madhya Pradesh topped the list
with the highest percentage of people who are living in multidimensional poverty.
Hence statement 1 is correct.
• Uttar Pradesh registered the largest decline in number of poor people escaping
multidimensional poverty. Hence statement 2 is correct.
.
IASBABA’S PRELIMS TEST SERIES (AIPTS) 2024 – CURRENT AFFAIRS (JULY 2023) SOLUTIONS

• The rural areas witnessed the fastest decline in multidimensional poverty than urban
areas.
• In rural areas, it saw a decline from 32.59% to 19.28% whereas in urban areas, it saw
a decline from 8.65% to 5.27%. Hence statement 3 is correct.

Source:
https://epaper.the
hindu.com/ccidist-
ws/th/th_delhi/issu
es/44078/OPS/GA6
BGEL8P.1.png?crop
FromPage=true

Q.58) Solution (b)

Statement Analysis:

• Pre-arrest Bail is also called anticipatory bail. It is a legal provision that allows an
accused person to apply for bail before being arrested.
• It is granted under section 438 of the Code of Criminal Procedure of 1973. Hence
statement 1 is correct.
• It is issued only by the Sessions Court and High Court. Hence statement 2 is
incorrect.
• The provision of pre-arrest bail is discretionary, and the court may grant bail after
considering the nature and gravity of the offence, the antecedents of the accused,
and other relevant factors.
• The person can seek pre-arrest bail only for non-bailable offences. Hence statement
3 is correct.
• The court may also impose a monetary bond, which the person seeking anticipatory
bail will have to pay if they fail to appear before the court or violate the conditions
imposed.

Source: https://epaper.thehindu.com/ccidist-
ws/th/th_delhi/issues/44078/OPS/GD6BGE0V2.1.png?cropFromPage=true
.
IASBABA’S PRELIMS TEST SERIES (AIPTS) 2024 – CURRENT AFFAIRS (JULY 2023) SOLUTIONS

Q.59) Solution (a)

Statement Analysis:

• The Local Currency Settlement System (LCSS) is a cross-border payment mechanism


that allows trading entities from two countries to make payments in their own
national currencies. It aims at putting in place a form of payment arrangement for
trading partner countries of India to raise their invoices and receive payments in
their respective home currency. Hence statement 2 is correct.
• LCSS will enable exporters and importers of both countries to pay in their respective
domestic currencies. So trade between countries with LCSS won’t be affected due to
sanctions and global political issues. Hence statement 1 is correct.

Source: https://epaper.thehindu.com/ccidist-
ws/th/th_delhi/issues/44078/OPS/G0HBGEJVB.1.png?cropFromPage=true

Q.60) Solution (c)

Statement Analysis:

• Kuki tribes are ethnic groups present in all Northeast Indian states except Arunachal
Pradesh, Myanmar, and Bangladesh. Hence statement 1 is correct.
• Lal is known as their village chief and has their own customary laws. Hence
statement 2 is correct.
• They do not marry outside their community and a majority of them follow
Christianity as their religion.
• They resisted British hegemony and started a rebellion before India’s independence.
• Kuki Rebellion took place during 1917 and they were defeated by the British in 1919.
Hence statement 3 is correct.

Source: https://epaper.thehindu.com/ccidist-
ws/th/th_delhi/issues/44078/OPS/G0HBGEJT9.1.png?cropFromPage=true

Q.61) Solution (b)

Statement Analysis:

• Black Sea Grain Initiative was set up to resume vital food and fertilizer exports from
Ukraine to the rest of the world. Hence statement 1 is correct.
• It allowed exports of grain, other foodstuffs, and fertilizer, including ammonia, to
resume through a safe maritime humanitarian corridor from three key Ukrainian
ports - Chornomorsk, Odesa, and Yuzhny/Pivdennyi, to the rest of the world.
.
IASBABA’S PRELIMS TEST SERIES (AIPTS) 2024 – CURRENT AFFAIRS (JULY 2023) SOLUTIONS

• It was brokered between Russia and Ukraine by the United Nations and Turkey.
Hence statement 2 is incorrect.
• It is implemented by the Joint Coordination Centre (JCC) which is hosted in Istanbul.
• JCC includes representatives from Russia, Türkiye, Ukraine, and the United
Nations. Hence statement 3 is correct.

IMAGE
SOURCE: U
N

Source:
https://epaper.
thehindu.com/
ccidist-
ws/th/th_delhi
/issues/44078/
OPS/G0HBGEJ
MP.1.png?crop
FromPage=true

Q.62) Solution (d)

Statement Analysis:

Context: Donanemab, if approved will become the second drug used to treat Alzheimer’s
disease.

• Leqembi is a drug approved for Alzheimer’s disease that targets the fundamental
pathophysiology of the disease. Hence option d is correct.
• Alzheimer’s disease is an irreversible, progressive brain disorder that slowly destroys
memory and thinking skills and, eventually, the ability to carry out simple tasks.

Source: https://epaper.thehindu.com/ccidist-
ws/th/th_delhi/issues/44078/OPS/G0HBGEJM3.1.png?cropFromPage=true

Q.63) Solution (c)

Statement Analysis:

• Electronic Bank Realization Certificate (e-BRC) is proof of export issued by banks. Any
firm applying for benefits under Foreign Trade Policy is required to furnish valid BRC
.
IASBABA’S PRELIMS TEST SERIES (AIPTS) 2024 – CURRENT AFFAIRS (JULY 2023) SOLUTIONS

as proof of realization of payment against exports made. Hence statement 1 is


correct.
• A bank issues it as confirmation that the exporter has received payment from the
buyer against the export of goods or services.
• It is an essential source of financial information and economic indicator.
• An exporter needs an eBRC to avail of the various export incentives (duty
exemptions, subsidies, low-cost loans, etc.) offered by the government as part of
its Foreign Trade Policy (FTP). Hence statement 2 is correct.

Source: https://epaper.thehindu.com/ccidist-
ws/th/th_delhi/issues/44078/OPS/G0HBGEJJ8.1.png?cropFromPage=true

Q.64) Solution (c)

Statement Analysis:

• The Enforcement Directorate (ED) is a multi-disciplinary organization that works


under the Department of Revenue, Ministry of Finance. Hence statement 1 is
correct.
• It is a financial investigation agency mandated with the investigation of offences
of money laundering and violations of foreign exchange laws.
• It enforces the provisions of the Foreign Exchange Management Act (FEMA) of 1999,
the Conservation of Foreign Exchange and Prevention of Smuggling Activities Act
(COFEPOSA) of 1974, the Fugitive Economic Offenders Act (FEOA) of 2018, and the
Prevention of Money Laundering Act of (PMLA) 2002. Hence statement 2 is correct.
• Its officers are recruited from various investigation agencies like Indian Revenue
Services (IRS), Indian Police Services (IPS), Excise Officers, Customs Officers, and
Income Tax Officers. Hence statement 3 is correct.

Source: https://epaper.thehindu.com/ccidist-
ws/th/th_delhi/issues/44198/OPS/GFPBGIHOB.1.png?cropFromPage=true

Q.65) Solution (b)

Statement Analysis:

• Steel Slag Road Technology uses the waste produced during steel production to build
robust and durable roads. Hence statement 1 is correct.
• It is an eco-friendly approach to managing industrial steel waste.
• It reduces the reliance on natural resources and is sustainable in the long term. It
does not depend on natural ballast and aggregates and hence does not deplete
natural resources. Hence statement 2 is incorrect.
.
IASBABA’S PRELIMS TEST SERIES (AIPTS) 2024 – CURRENT AFFAIRS (JULY 2023) SOLUTIONS

• India is the world's second-largest steel-producing country, generating around 19


million tonnes of steel slag as solid waste. It is projected to increase to a staggering
60 million tonnes by 2030, with each tonne of steel production resulting in about
200 kg of steel slag waste.
• It plays a very important role in fulfilling the mission of waste to wealth in India.
Hence statement 3 is correct.

Source: https://pib.gov.in/PressReleseDetail.aspx?PRID=1940792

Q.66) Solution (a)

Statement Analysis:

• Bhoomi Samman Awards aims to recognise outstanding performance in the


implementation of the Digital India Land Records Modernization Programme
(DILRMP). Hence statement 1 is correct.
• If a district has completed 100% of the components of the DILRMP, it will be given
platinum grading.
• It was launched in 2023 and is presented by the President of India at Vigyan
Bhawan, New Delhi.
• DILRMP is a central sector scheme being implemented by the Department of Land
Resources under the Ministry of Rural Development. Hence statement 2 is
incorrect.
• Its components include the computerization of land records, surveys, and
registration.
• It attempts to build upon the commonalities that exist in the arena of land records in
various States to develop an appropriate Integrated Land Information Management
System (ILIMS) across the country.

Source: https://newsonair.gov.in/News?title=President-Murmu-to-present-
%26%2339%3bBhoomi-Samman%26%2339%3b-2023-on-July-18-in-New-Delhi&id=464326

Q.67) Solution (b)

Statement Analysis:

• Bay of Bengal Initiative for Multi-Sectoral Technical and Economic Cooperation


(BIMSTEC) is a regional organization that was established by the Bangkok Declaration
on 6th June 1997. Hence statement 1 is correct.
.
IASBABA’S PRELIMS TEST SERIES (AIPTS) 2024 – CURRENT AFFAIRS (JULY 2023) SOLUTIONS

• It has seven
member countries of
which five member
countries – India,
Bhutan, Bangladesh,
Nepal, and Sri Lanka
are from South Asia
and two countries –
Myanmar and
Thailand are from
Southeast Asia.
Hence statement 2 is
incorrect.
• Its secretariat
is in Dhaka,
Bangladesh. Hence statement 3 is correct.

Source: https://newsonair.gov.in/News?title=Food%2c-health-and-energy-security-are-
common-concerns-for-BIMSTEC-countries%3a-S.-Jaishankar&id=464383

Q.68) Solution (b)


Statement Analysis:

• Khashaba Dadasaheb Jadhav is an Indian wrestler who is nicknamed ‘Pocket


Dynamo’. Hence statement 1 is correct.
• He was born in a village in Maharashtra.
• He was the first athlete from independent India to win an individual medal in the
Olympics. Hence statement 2 is correct.
• He is known for winning a bronze medal at the 1952 Summer Olympics in Helsinki.
• He provided shelter to revolutionaries and circulated letters against the British
during the Quit India Movement. Hence statement 3 is incorrect.
.
IASBABA’S PRELIMS TEST SERIES (AIPTS) 2024 – CURRENT AFFAIRS (JULY 2023) SOLUTIONS

Source:
https://twitter.co
m/MIB_India/stat
us/168304028238
2614528

Q.69) Solution (c)

Statement Analysis:

• Gingee Venkataramana Temple was built in the 16th century by Muthyala Nayaka in
Tail Nadu. Hence statement 1 is correct.
• Its entrance gateway depicts Ramayana scenes, Vishnu's incarnations, and the
Samudra Manthan. Hence statement 2 is correct.
• Its premises have a tank called Chettikulam which is fed by perennial springs and do
not dry even in summer.
• It was built either in the 17th century by Rama Chetti or in the 18th century by Rama
Shetty under the Maratha Kingdom. Hence statement 3 is correct.
• To the east of the temple is the Kalyanamahal, an architectural treasure built in
the Vijayanagara style, is made up of a square court surrounded by rooms with
verandahs on arches with stairways,
in the middle of which is an 8-storey
square tower with pyramidal roof.
.
IASBABA’S PRELIMS TEST SERIES (AIPTS) 2024 – CURRENT AFFAIRS (JULY 2023) SOLUTIONS

Source: https://indianculture.gov.in/node/2730057

Q.70) Solution (d)

Statement Analysis:

Ramgarh Vishdhari Tiger Reserve is the fourth tiger reserve in Rajasthan, India. It
comprises hilly dry deciduous forests consisting of Dhok, Khair, and Salar on Vindhyan
formations. It plays a critical role in the movement of tigers. It has culturally significant
monuments like Bhimlat. Hence option d is correct.

IMAGE SOURCE: firstpost.com

Source: https://www.hindustantimes.com/cities/jaipur-news/rajasthan-tigress-spotted-
with-three-cubs-in-ramgarh-visdhari-reserve-101689513175282.html
.
IASBABA’S PRELIMS TEST SERIES (AIPTS) 2024 – CURRENT AFFAIRS (JULY 2023) SOLUTIONS

Q.71) Solution (b)

Statement Analysis:

Intercontinental Ballistic Missile Country


(ICBM)
1. Dongfeng-41 China

2. RS-24 Yars Russia

3. Trident II United States

4. Hwasong-18 North Korea

Hence option b is correct.

Source: https://indianexpress.com/article/explained/explained-global/north-korea-test-
hwasong-18-new-missile-significance-8841498/

Q.72) Solution (b)

Statement Analysis:

• The Commission on Genetic Resources for Food and Agriculture (CGRFA) is the only
permanent intergovernmental body focused on conserving all types of biodiversity
for food and agriculture. Hence statement 1 is correct.
• It offers a unique platform for its members and other stakeholders to promote a
world without hunger by fostering the use and development of the whole portfolio
of biodiversity important to food security and rural poverty.
• Its membership is open to all members of the Food and Agriculture Organization
(FAO). Hence statement 2 is incorrect.
• It consists of 179 Member States and the European Union.
• It meets regularly to address policies on genetic resources for food and agriculture.
• It focuses on the state of the world’s forest and plant genetic resources, access and
benefit-sharing policies, and biotechnologies for the conservation, and sustainable
use of genetic resources. Hence statement 3 is correct.

Source: https://www.downtoearth.org.in/news/wildlife-biodiversity/genetic-resources-
commission-gathers-in-rome-to-deliberate-on-biodiversity-nutrition-human-health-90700

Q.73) Solution (d)

Statement Analysis:

Gambusia affinis is a freshwater fish. It is also known as mosquito fish and has been a part
of mosquito-control strategies for over a century in various parts of the world, including
.
IASBABA’S PRELIMS TEST SERIES (AIPTS) 2024 – CURRENT AFFAIRS (JULY 2023) SOLUTIONS

India. A single full-grown fish eats about 100 to 300 mosquito larvae per day. Hence option
d is correct.

Source: https://www.downtoearth.org.in/news/wildlife-biodiversity/gambusia-this-
solution-could-actually-be-an-invasive-problem-90665

Q.74) Solution (b)

Statement Analysis:

• External commercial borrowing (ECB) is borrowing made in foreign currency by non-


resident lenders to Indian borrowers. It is an instrument used in India to facilitate
Indian companies to raise money outside the country in foreign currency.
• It provides an opportunity to borrow large volumes of funds available for a relatively
long term. Hence statement 1 is correct.
• The cost of funds is usually cheaper from external sources if borrowed from
economies with a lower interest rate.
• Higher debt on the company’s balance sheet is usually viewed negatively by the
rating agencies, resulting in a possible downgrade by rating agencies which
eventually might increase the cost of debt. It is one of the disadvantages of ECBs.
Hence statement 2 is incorrect.
• It is just a form of a loan and may not be of an equity nature or convertible to equity.
Hence, it does not dilute the stake in the company and can be done without giving
away control because debtors do not enjoy voting rights. Hence statement 3 is
correct.

IMAGE

SOURCE: efinancemanagement.com
.
IASBABA’S PRELIMS TEST SERIES (AIPTS) 2024 – CURRENT AFFAIRS (JULY 2023) SOLUTIONS

Source: https://www.financialexpress.com/economy/big-jump-in-ecbs-signals-revival-of-
private-capex/3175122/

Q.75) Solution (c)

Statement Analysis:

• The Gomti River is a tributary of the Ganga River. Hence statement 1 is incorrect.
• It originates near Mainkot, from Gomat Taala Lake, Pilibhit. Hence statement 2 is
correct.
• Lucknow, the capital of Uttar Pradesh is located on its banks. Hence statement 3 is
correct.
• The River flows through Sitapur, Lucknow, Barabanki, Sultanpur, and
Jaunpur before meeting the Ganga River.
• It meets the Ganges River at Kaithi, Ghazipur district. Hence statement 4 is correct.

IMAGE
SOURCE: ResearchGate

Source: https://www.downtoearth.org.in/news/urbanisation/dte-exclusive-2020-order-on-
floodplain-zoning-declaring-gomti-non-perennial-river-draws-flak-90655

Q.76) Solution (a)

Statement Analysis:

• Advance Authorisation Scheme helps in making India’s products competitive in the


global market. Hence statement 1 is correct.
• It was introduced by the Government of India under the Foreign Trade Policy 2015-
2020.
.
IASBABA’S PRELIMS TEST SERIES (AIPTS) 2024 – CURRENT AFFAIRS (JULY 2023) SOLUTIONS

• It is implemented by the Directorate General of Foreign Trade (DGFT).


• Advance Authorization Scheme exempts the basic customs duty, education cess,
social welfare cess, anti-dumping duty, countervailing duty, and safeguard duties.
Hence statement 2 is correct.

Source: https://pib.gov.in/PressReleasePage.aspx?PRID=1940171

Q.77) Solution (c)

Statement Analysis:

• Bosporus Strait connects the Black Sea to the Sea of Marmara. Hence statement 1 is
correct.
• The Strait of the Dardanelles connects the Sea of Marmara to the Aegean Sea. Hence
statement 2 is correct.
• Kerch Strait connects the Black Sea to the Sea of Azov. Hence statement 3 is correct.

IMAGE SOURCE: World Atlas

IMAGE SOURCE: Britannica


.
IASBABA’S PRELIMS TEST SERIES (AIPTS) 2024 – CURRENT AFFAIRS (JULY 2023) SOLUTIONS

IMAGE SOURCE: Britannica

Source: https://newsonair.gov.in/News?title=India-supports-UN-efforts-to-continue-key-
Black-Sea-Grain-deal-terminated-by-Russia&id=464484

Q.78) Solution (b)

Statement Analysis:

• The Index of Industrial Production (IIP) is published monthly by the National


Statistical Office (NSO) under the Ministry of Statistics and Programme
Implementation. Hence statement 1 is incorrect.
• It indicates the relative change over time in the volume of production in the
industrial sector. Hence statement 2 is correct.
• It is an effective tool to measure the trend of current industrial production that is
used by government agencies including the Ministry of Finance, the Reserve Bank of
India, etc., for policy-making purposes.
• The eight core sector industries in decreasing order of their weightage: Refinery
Products> Electricity> Steel> Coal> Crude Oil> Natural Gas> Cement> Fertilizers.
• Refinery products constitute the highest percentage among the eight core industries.
Hence statement 3 is correct.

Source: https://newsonair.gov.in/News?title=IIP-grows-by-5.2%25-in-May-2023-from-
4.2%25-in-April-2022&id=464119
.
IASBABA’S PRELIMS TEST SERIES (AIPTS) 2024 – CURRENT AFFAIRS (JULY 2023) SOLUTIONS

Q.79) Solution (c)

Statement Analysis:

• Gharials are an indicator species found in the tributaries of River Ganga.


• They are an indicator of clean water. They are found in three tributaries of the
Ganga River - the Chambal and the Girwa Rivers in India and the Rapti-Naryani
River in Nepal. Hence statement 1 is correct.
• They are listed as critically endangered on the IUCN Red List and in Appendix I of the
CITES. Hence statement 2 is correct.
• They are protected under Schedule I of the Wildlife Protection Act of 1972. Hence
statement 3 is correct.

Source: https://www.thehindu.com/news/national/americas-largest-reptile-sanctuary-
wants-to-import-gharials-from-india/article67093219.ece

Q.80) Solution (c)

Statement Analysis:

• Rudragiri Hillock has prehistoric rock paintings from the Mesolithic period and art
from the Kakatiya dynasty is located in Andhra Pradesh. Hence statement 1 is
incorrect.
• It is nestled amidst the Eastern Ghats and features five naturally formed rock
shelters at its foothills, facing westward.
• These shelters served as living quarters for people during the Mesolithic age around
5000 B.C.
• It is the first cave representing murals of battle between the Vanara brothers, Vali,
and Sugriva. Hence statement 2 is correct.
• Ganapati Deva Maharaja (1199-1262 AD) who was built the Muppavaram temple
and a prominent figure of the Kakatiya dynasty is likely to be the patron of the rich
ancient mural heritage found at Rudragiri.
• It has a sketch of Hanuman, accompanied by sacred symbols of the conch and the
fire altar. Hence statement 3 is correct.

Note:
Kakatiya kingdom was a south Indian dynasty that ruled most of the eastern Deccan region
comprising present-day Telangana and Andhra Pradesh, and parts of eastern Karnataka and
southern Odisha. They ruled between the 12th and 14th centuries.

Source: https://www.thehindu.com/news/national/andhra-pradesh/a-fascinating-fusion-of-
rock-art-at-rudragiri-hillock/article67092853.ece
.
IASBABA’S PRELIMS TEST SERIES (AIPTS) 2024 – CURRENT AFFAIRS (JULY 2023) SOLUTIONS

Q.81) Solution (b)

Statement Analysis:

• The Prime Minister Street Vendor’s AtmaNirbhar Nidhi (PM SVANidhi) Scheme
provides collateral-free working capital loans up to INR 10,000/- to street vendors
for one year tenure. Hence statement 1 is incorrect.
• Street vendors with a Certificate of Vending/Identity Card issued by Urban Local
Bodies (ULBs) are eligible for the scheme.
• It is implemented by the Small Industries Development Bank of India (SIDBI). Hence
statement 2 is correct.
• It encourages entrepreneurship and creates employment opportunities.
• An interest subsidy of 7% per annum is provided on timely/ early repayment of the
loan. Hence statement 3 is correct.
• There is no processing fee to apply for the scheme, making it cost-effective for
street vendors.

IMAGE SOURCE: makspay.com

Source: https://newsonair.gov.in/News?title=Secretary%2c-DFS%2c-holds-review-meeting-
with-heads-of-Public-Sector-Banks-to-assess-progress-under-PM-SVANidhi-
scheme&id=464616

Q.82) Solution (c)

Statement Analysis:

• Picolinic acid is a pyridinemonocarboxylic acid i.e., the carboxy group is located at


position 2. Hence statement 1 is correct.
.
IASBABA’S PRELIMS TEST SERIES (AIPTS) 2024 – CURRENT AFFAIRS (JULY 2023) SOLUTIONS

• It is an intermediate in the metabolism of tryptophan, an essential amino acid used


to make proteins.
• It helps in the absorption of zinc from our gut. Hence statement 2 is correct.
• It stays inside the body only for a short duration and is usually excreted out quickly.
• It disrupts the entry of enveloped viruses into the host’s cell and prevents infection.
Hence statement 3 is correct.
• It leads to an increase in the number of immune cells in the animals and is effective
against a variety of enveloped viruses, including flaviviruses like the Zika virus and
the Japanese encephalitis virus.

Source: https://www.thehindu.com/features/november-fest/frnf-bangalore/iisc-study-
reveals-that-picolinic-acid-can-block-viruses-causing-sars-cov-2-and-influenza-
a/article67093012.ece

Q.83) Solution (d)


Statement Analysis:

Animal Husbandry Infrastructure Development Fund (AHIDF) was launched in 2021 by the
Ministry of Fisheries, Animal Husbandry and Dairying. It is implemented by the Department
of Animal Husbandry & Dairying. Its objectives are:
• To help increasing of milk and meat processing capacity and product diversification
thereby providing greater access for unorganized rural milk and meat producers to
organized milk and meat market.
• To make available increased price realization for the producer.
• To make available quality milk and meat products for the domestic consumer.
• To fulfill the objective of protein-enriched quality food requirement of the growing
population of the country and prevent malnutrition in one of the highest
malnourished children population in the world.
• Develop entrepreneurship and generate employment.
• To promote exports and increase the export contribution in the milk and meat
sector.
• To make available quality concentrated animals feed to the cattle, buffalo, sheep,
goat, pig, and poultry to provide balanced ration at affordable prices.

The activities are eligible for benefits under the Animal Husbandry Infrastructure
Development Fund (AHIDF) are:
• Dairy Processing
• Meat Processing
• Animal Feed Manufacturing
• Breed Improvement Technology
• Veterinary Vaccine and Drugs Production
.
IASBABA’S PRELIMS TEST SERIES (AIPTS) 2024 – CURRENT AFFAIRS (JULY 2023) SOLUTIONS

• Animal Waste to Wealth Management


Hence option d is correct.

Source: https://www.business-standard.com/economy/news/first-ever-credit-guarantee-
scheme-for-livestock-sector-details-here-123072000479_1.html

Q.84) Solution (a)

Statement Analysis:

• Bacteriophages are viruses that infect and replicate only in bacterial cells. Hence
statement 1 is correct.
• These viruses kill bacteria in our microbiomes.
• They only attack bacteria and are harmless to people, animals, and plants.
• They are classified in a number of virus families some examples include Inoviridae,
Microviridae, Rudiviridae, and Tectiviridae.
• They are clinically relevant for their ability to distinguish strains of the same bacterial
species.
• They are vectors for horizontal gene transfer, including antimicrobial resistance
genes. Hence statement 2 is correct.
• They are found in soil, water, and sewage. Hence statement 3 is incorrect.
• Like all viruses, phages are simple organisms that consist of a core of genetic
material (nucleic acid) surrounded by a protein capsid.
• Capsid of a bacteriophage can be icosahedral, filamentous, or head-tail in shape.
• The head-tail structure seems to be unique to phages and their close relatives.

Source: https://www.hindustantimes.com/lifestyle/health/bacteriophages-the-good-
viruses-that-fight-bacteria-101689831489217.html

Q.85) Solution (b)

Statement Analysis:

• DPT3 Vaccine provides protection against three severe bacterial diseases -


Diphtheria, Tetanus, and Pertussis. Hence statement 1 is incorrect.
• Diphtheria causes a thick covering in the back of the throat. It can lead to breathing
problems, paralysis, heart failure, and even death in severe cases.
• Tetanus causes painful tightening of the muscles usually all over the body. It can lead
to locking of the jaw as a result the victim cannot open his mouth or swallow. Deaths
because of Tetanus are around 10%.
• Pertussis (whooping cough or black cough) causes severe coughing spells that it is
hard for infants to eat, drink or even breathe. These spells can last for weeks. It can
lead to pneumonia, convulsions, brain damage, and death.
.
IASBABA’S PRELIMS TEST SERIES (AIPTS) 2024 – CURRENT AFFAIRS (JULY 2023) SOLUTIONS

• All children should get five doses of the DTP vaccine at 6 weeks, 10 weeks, 14 weeks,
16-18 months, and 5 years. Hence statement 2 is correct.
• A child who had a life-threatening allergic reaction after a dose of DTP vaccination
should not get another dose. Hence statement 3 is correct.
• A child who had encephalopathy (brain illness) or nervous system disease within 7
days after a dose of DTP should not get another dose.
• A child who had a temperature of 105oF within 48 hours after a dose of DTP should
probably not get another dose of the Pertussis-containing vaccine.
• A child who collapses or goes into a "shock-like" state within 48 hours after a dose of
DTP should probably not get another dose of the Pertussis-containing vaccine.
• A child who cries continuously for 3 or more hours within 48 hours after a dose of
DTP should probably not get another dose of the Pertussis-containing vaccine.
• A child who has convulsions within 3 days after a dose of DTP should probably not
get another dose of the Pertussis-containing vaccine.

Source: https://www.thehindu.com/news/national/india-recorded-all-time-high-of-93-dpt3-
immunisation-coverage-in-2022-who/article67094235.ece

Q.86) Solution (b)

Statement Analysis:

The benefits of hydrogen as a fuel:


• It is environmentally friendly and non-toxic.
• It doesn’t produce emissions as other fossil fuels.
• It is three times as powerful as gasoline and other fossil fuels.
• It is more efficient than diesel or gas.

Disadvantages of hydrogen fuel:


• It does not occur in deposits or reserves like fossil fuel.
• It is produced using chemical reactions.
• It is highly flammable.
• It is difficult to store.
Hence option b is correct.

Source: https://www.downtoearth.org.in/news/energy-efficiency/g20-energy-ministers-
call-for-cooperation-on-nuclear-energy-low-emission-hydrogen-90777

Q.87) Solution (b)

Statement Analysis:
.
IASBABA’S PRELIMS TEST SERIES (AIPTS) 2024 – CURRENT AFFAIRS (JULY 2023) SOLUTIONS

• Malware is any kind of software that is designed to cause damage to a single


computer, server, or computer network. Hence statement 1 is correct.
• Spoofing involves impersonating a bank’s URL with a website that is quite similar to
the original one and has similar functions as well. Hence statement 2 is correct.
• Phishing is attempting to obtain sensitive information for fraudulent activities, by
disguising oneself as an authentic entity via electronic communication. Hence
statement 3 is incorrect.
• Denial of Service is an attack meant to shut down a machine or network, making it
inaccessible to its intended users. Hence statement 4 is incorrect.
• Man-in-the-middle (MitM) attacks occur when attackers insert themselves into a
two-party transaction. Once the attackers interrupt the traffic, they can filter and
steal data.
• Structured Query Language (SQL) Injection is a programming language used to
communicate with databases. Many of the servers that store critical data for
websites and services use SQL to manage the data in their databases. A SQL injection
attack specifically targets such kinds of servers, using malicious code to get the
server to divulge information it normally would not.

Source: https://newsonair.gov.in/News?title=Gravity-of-cyber-risks-will-increase-with-
advent-of-disruptive-technologies-such-as-AI%3a-NSA-Ajit-Doval-at-BRICS-
Meeting&id=464835

Q.88) Solution (d)

Statement Analysis:

The Chicago Convention of 1944 is related to the International Civil Aviation Organization
(ICAO). It is a United Nations agency, established to help countries share their skies to their
mutual benefit. It assists the 193 Contracting States to the Chicago Convention as they
cooperate here to adopt standards, practices, and policies for international civilian flight.
Industry and civil society groups, in addition to relevant multilateral organizations,
contribute importantly to these ICAO outcomes as ‘Invited Organizations’. India is one of the
founding members of ICAO. It is headquartered in Quebec, Canada. Hence option d is
correct.

Source: https://newsonair.gov.in/News?title=DGCA-adopts-guidelines-for-environmental-
protection-developed-by-ICAO-to-reduce-greenhouse-gas-emissions%3a-Govt-in-
RS&id=464814
.
IASBABA’S PRELIMS TEST SERIES (AIPTS) 2024 – CURRENT AFFAIRS (JULY 2023) SOLUTIONS

Q.89) Solution (a)

Statement Analysis:

• India Climate Energy Dashboard (ICED) 3.0 is India’s one-stop platform for near real-
time data on the energy sector, climate, and related economic datasets based on
government-published sources. Hence statement 1 is correct.
• It provides near real-time data with in-built analytics to track climate action
progress.
• It enables users to freely access and analyze datasets using an analytical engine and
facilitate insights and enhance understanding of the energy and climate sectors.
• It is released by National Institution for Transforming India (NITI Aayog) in
collaboration with the energy and climate think-tank Vasudha Foundation. Hence
statement 2 is incorrect.

Source: https://pib.gov.in/PressReleaseIframePage.aspx?PRID=1941095

Q.90) Solution (b)

Statement Analysis:

• National Dairy Development Board (NDDB) is a statutory body initially registered as a


society under the Societies Act 1860. It was later merged with the former Indian
Dairy Corporation, which was created and registered under the Companies Act 1956,
by the NDDB Act 1987, which took effect on October 12, 1987. Hence statement 1 is
correct.
• Its objective is to replace exploitation with empowerment, tradition with modernity,
stagnation with growth, transforming dairying into an instrument for the
development of India’s rural people.
• It works under the Ministry of Fisheries, Animal Husbandry, and Dairying. Hence
statement 2 is incorrect.
• It launched the world’s largest dairy development program, Operation Flood. It has
helped dairy farmers direct their own development, placing control of the resources
they create in their own hands. Hence statement 3 is correct.
• The bedrock of Operation Flood has been village milk producers' cooperatives, which
procure milk and provide inputs and services, making modern management and
technology available to members. Operation Flood's objectives included :
✓ Increase milk production ("a flood of milk")
✓ Augment rural incomes
✓ Reasonable prices for consumers
• A National Milk Grid links milk producers throughout India with consumers in over
700 towns and cities, reducing seasonal and regional price variations while ensuring
that the producer gets fair market prices in a transparent manner on a regular basis.
.
IASBABA’S PRELIMS TEST SERIES (AIPTS) 2024 – CURRENT AFFAIRS (JULY 2023) SOLUTIONS

Source: https://timesofindia.indiatimes.com/education/news/international-symposium-on-
sustainable-livestock-transformation-kicks-off-at-nddb/articleshow/101949651.cms

Q.91) Solution (c)

Statement Analysis:

An adjournment motion is introduced in the Parliament to draw the attention of the


House to a definite matter of urgent public importance.
Restrictions for moving an adjournment motion are:
• It must not cover more than one matter. Hence statement 1 is correct.
• It must be restricted to a specific matter of recent occurrence. Hence statement 2 is
correct.
• It must not raise a question of privilege. Hence statement 3 is correct.
• It must not raise a question of privilege.
• It must not revive discussion on a matter that has been discussed in the same
session.
• It must not deal with any matter that is under adjudication of court.
• It must not raise any question that can be raised on a distinct motion.

Source: https://indianexpress.com/article/explained/everyday-explainers/adjournment-
motion-manipur-congress-mp-8854464/

Q.92) Solution (d)

Statement Analysis:

• Silver cockscomb is a weed that attracts insects, caterpillars, worms, and moths that
can harm crops. Hence statement 1 is correct.
• It is a short-lived 50-60 cm-tall plant that bears simple, spirally arranged leaves
around the stem with pinkish or silky white flowers.
• It is used as a nutritious leafy green vegetable by the Soliga tribe.
• The women of the Soliga tribe collect edible leaves and young shoots of silver
cockscomb to prepare a mash called massanne, which is eaten just before the
monsoon (April to June) to help lower the body's heat and reduce stomach burn,
which often stems from indigestion. Hence statement 2 is correct.
• It can grow well even on fallow land and in drought-like conditions.
• It belongs to the Amaranthaceae family, which includes economically important
plants like spinach and beetroot. Hence statement 3 is correct.

Source: https://www.downtoearth.org.in/news/food/green-promise-silver-cockscomb-isn-t-
a-troublesome-weed-for-karnataka-s-soliga-tribe-90593
.
IASBABA’S PRELIMS TEST SERIES (AIPTS) 2024 – CURRENT AFFAIRS (JULY 2023) SOLUTIONS

Q.93) Solution (c)

Statement Analysis:

Ludwigia peruviana is an invasive aquatic weed. It is originally native to Central and South
America. It is an aquatic plant, which is now challenging the existence of local vegetation in
various swampy areas around the world. It is a perennial shrub that is deciduous in cooler
climates and evergreen in warmer tropical climates. Hence option c is correct.

Source: https://www.thehindu.com/sci-tech/energy-and-environment/peruvian-weed-
threatens-elephant-habitats-in-tamil-nadu/article67109844.ece

Q.94) Solution (c)

Statement Analysis:

• Malati Devi Choudhury (26 July 1904– 15 March 1998) was an Indian civil rights and
freedom activist and Gandhian.
• Malati Devi Choudhury organised the ‘Krusaka Andolana’ as part of the freedom
struggle against the zamindars and moneylenders. Hence statement 1 is correct.
• She organised the Utkal Congress
Samajvadi Karmi Sangh, which later
became the Orissa Provincial Branch of
the All India Congress Socialist Party and
the Bajiraut Chhatravas, had its genesis in
the Prajamandal Movement (the
resistance movement organized and
sustained by the people) and its initial
activities were geared towards providing
residential facilities and educational
opportunities to the children of the
freedom fighters. Hence statement 2 is
correct.

Source: https://amritmahotsav.nic.in/unsung-heroes-detail.htm?297
.
IASBABA’S PRELIMS TEST SERIES (AIPTS) 2024 – CURRENT AFFAIRS (JULY 2023) SOLUTIONS

Q.95) Solution (b)

Statement Analysis:

• African Union is a successor to the Organization of African Unity (OAU, 1963-1999).


Hence statement 1 is correct.

Objectives of AU:
✓ Achieve greater unity and solidarity between African countries and their people.
✓ Defend the sovereignty, territorial integrity, and independence of its Member States.
✓ Accelerate the political and socio-economic integration of the continent.
✓ Encourage international cooperation.
✓ Promote peace, security, and stability on the continent.
✓ Promote democratic principles and institutions, popular participation, and good
governance.
✓ Work with relevant international partners in the eradication of preventable diseases
and the promotion of good health on the continent.
✓ Ensure the effective participation of women in decision-making, particularly in the
political, economic, and socio-cultural areas.
✓ Develop and promote common policies on trade, defense, and foreign relations.
• All its 55 member countries are only from the African continent. Hence statement 2
is correct.
• It is headquartered in Addis Ababa, Ethiopia. Hence statement 3 is incorrect.

Source: https://www.thehindu.com/news/national/overwhelming-response-to-pms-
proposal-of-making-african-union-permanent-member-of-g-20-amitabh-
kant/article67112879.ece

Q.96) Solution (b)

Statement Analysis:

A new recent study confirmed that the intensity of the system of ocean currents that
regulates our climate is decreasing from year to year. According to the study, the Arctic is
warming more than four and a half times faster than the rest of the planet.

• Gulf Stream is a strong warm ocean current. Hence statement 1 is incorrect.


• It is part of a general clockwise-rotating system of currents in the North Atlantic and
is fed by the westward-flowing North Equatorial Current moving from North Africa to
the West Indies.
• It is primarily formed by the convergence of warm waters from the Caribbean Sea
and the Gulf of Mexico.
• It then travels northward along the eastern coast of the United States.
.
IASBABA’S PRELIMS TEST SERIES (AIPTS) 2024 – CURRENT AFFAIRS (JULY 2023) SOLUTIONS

• It carries warm water from the Gulf of Mexico into the Atlantic Ocean. Hence
statement 2 is correct.
• It extends all the way up the eastern coast of the United States and Canada.

IMAGE SOURCE: scijinks

Source: https://www.domusweb.it/en/news/2023/07/31/gulf-stream-is-slowing-and-is-
likely-to-collapse-by-2025.html

Q.97) Solution (d)

Statement Analysis:

The Batagaika crater is the world’s largest permafrost crater located in Sakha Republic,
Russia. Hence option d is correct.

Source: https://www.downtoearth.org.in/video/climate-change/the-batagaika-crater-a-
warning-sign-for-the-earth-90815

Q.98) Solution (c)

Statement Analysis:

• The findings of the State of Climate in Asia 2022 Report released by the World
Meteorological Organisation (WMO):
• The number of disaster events in Asia decreased when compared to the previous
year. Over 100 disaster events took place in 2021 whereas 81 disaster events took
place in 2022. Hence statement 1 is correct.
• Pakistan followed by China and India suffered the most severe economic losses that
occurred due to floods in the Asia region. Hence statement 2 is correct.
• China suffered the most severe economic losses that occurred due to droughts in the
Asia region.
.
IASBABA’S PRELIMS TEST SERIES (AIPTS) 2024 – CURRENT AFFAIRS (JULY 2023) SOLUTIONS

Source: https://www.downtoearth.org.in/news/climate-change/extreme-continent-new-
wmo-report-paints-alarming-picture-of-climate-emergency-in-asia-90882

Q.99) Solution (b)

Statement Analysis:

Parachik Glacier is located in Ladakh, India. It is one of the largest glaciers in the Suru River
valley, part of the southern Zanskar Ranges in the western Himalayas. Hence option b is
correct.

Source: https://dst.gov.in/3-new-lakes-may-form-parkachik-glacier-ladakh-due-glacial-
retreat
https://indianexpress.com/article/india/ladakh-glacier-melting-may-form-three-lakes-says-
study-8865807/

Q.100) Solution (b)

Statement Analysis:

A joint expedition of ocean scientists from India, Bangladesh and Mauritius was organised
by the Indian National Centre for Ocean Information Services (INCOIS), Ministry of Earth
Sciences under the regional framework of Colombo Security Conclave (CSC) concluded on
24 Jul 2023.

• Colombo Security Conclave is a trilateral maritime security grouping among member


countries. Hence statement 1 is incorrect.
• It was formed in 2011.
• It is based on the following five pillars:
✓ Maritime Safety and Security
✓ Countering Terrorism and Radicalization
✓ Combating Trafficking and Transnational Organized Crime
✓ Cyber Security, Protection of Critical Infrastructure and Technology
✓ Humanitarian Assistance and Disaster Relief
✓ The 7th Deputy NSA level meeting of the Colombo Security Conclave (CSC) was
hosted by Maldives in July 2023.
• Its members are India, Sri Lanka, Maldives, and Mauritius. Hence statement 2 is
correct.
• Bangladesh and Seychelles attended as observers.

You might also like